CIC: Practice Exam 1

Ace your homework & exams now with Quizwiz!

17. What is the probability of committing a Type I error if the p value is 0.10? a. 1 in 10 b. 1 in 100 c. 1 in 5 d. 1 in 20

1 in 10 Rationale: A Type I error occurs when one rejects the null hypothesis (Ho) - when it is true. This is also called a false-positive result (we incorrectly conclude that the research hypothesis is true when in fact it is not). The p value or calculated probability, is the estimated probability of rejecting the null hypothesis of a study question when that hypothesis is true. A p value of 0.10 indicates a 10 percent (or 1 in 10) chance of making a Type 1 error. Reference: APIC Text, 4th edition, Chapter 13

6. Which of the following organisms have been associated with the transmission of infections after body piercing? 1) Atypical Mycobacterium species 2) Staphylococcus species 3) Pseudomonas species 4) Haemophilus species a. 1,2,3 b. 2,3,4 c. 1,3,4 d. 1,2,4

1, 2, 3 Not Haemophilus species Rationale: Body piercing activities can transmit infectious diseases. Bacterial infections may result from improper initial piercing technique or from poor hygiene. The organisms involved in most earlobe-piercing infections are often considered normal skin flora, including Staphylococcus and Streptococcus species. Higher ear piercings in the ear cartilage have been associated with more pathogenic organisms, including Pseudomonas species. Multiple cases of atypical Mycobacterium infections after piercing have been reported. Mastitis due to Mycobacterium abscesses has been reported after a nipple piercing; Mycobacterium flavescens has been reported after an eyebrow piercing, and Mycobacterium chelonae has been reported after a navel piercing. There are case reports of infective endocarditis that required treatment with intravenous antibiotics, and in some cases surgical intervention to replace heart valves; the infections resulting from navel, earlobe, and nipple piercings. There has also been a case report of tetanus after a naval piercing. Reference: APIC Text, 4th edition, Chapter 123 Body Piercing, Tattoos, and Electrolysis

124. Which of the following need to be considered when updating the annual infection risk assessment? 1) An evaluation of the previous year's goals and objectives 2) An identification of risks based on geographic location, community, and population served 3) Risks related to the type of services that the facility provides 4) A broad assessment of all risks identified in the facility a. 1, 2, 4 b. 1, 2, 3 c. 2, 3, 4 d. 1, 3, 4

1, 2, 3 Rationale: The risk assessment should be tailored to the risks in specific to a facility and must include identification of risks based on the facility's geographic location, community, and population served as well as the types of services the facility offers. The annual risk assessment should take into account the previous year's goals and objectives and whether the facility met the targets associated with those goals. Reference: APIC Text, 4th edition, Chapter 1

134. Which of the following parameters affect(s) the effectiveness of ethylene oxide (ETO) sterilization? 1) Gas concentration 2) Temperature 3) Relative humidity 4) Exposure time a. 1,2 b. 1, 3, 4 C. 1, 2:3, 4 d. 1,4

1, 2, 3,4 Rationale: Ethylene oxide (ETO) sterilization is affected by four parameters. The concentration of gas should be between 450 and 1200 mg/L, the temperature range should be between 37 and 63°C, the relative humidity should be between 40 and 80 percent, and the exposure time should be between and 6 hours.

98. Strategies to prevent infection of a dialysis access site include the following: 1) Utilize an arteriovenous (AV) graft if a fistula cannot be established 2) Use a tunneled cuffed catheter for acute dialysis if use will be more than 3 weeks 3) Subclavian access is preferred over jugular options 4) Use femoral catheters only in bedbound patients for no more than 5 days a. 1, 3, 4 b. 1, 2, 4 c. 2, 3, 4 d. 1, 2, 3

1, 2, 4 Rationale: The risk of infection related to vascular access for hemodialysis varies with the type of vascular access used. Risk for bacteremia increases sevenfold in patients with a dialysis catheter compared to those with a primary arteriovenous (AV) fistula. It is recommended that an AV fistula be created and used for long-term hemodialysis treatment because of the lower incidence of infection. If an AV fistula cannot be established, an AV graft is the next preferred type of access. Because of infection risk, creation of the fistula in the upper arm is preferred over the thigh. For acute hemodialysis, where access for less than 3 weeks duration is anticipated, vascular access may be obtained using a noncuffed or cuffed catheter. However, if a catheter must be used for access for longer than 3 weeks, a tunneled, cuffed venous catheter should be used. The preferred insertion site is the right internal jugular. Because there is a greater incidence of central venous thrombosis and stenosis when the subclavian is used, subclavian access should be used only when jugular options are not available and permanent vascular access is not required. In addition, tunneled cuffed catheters should not be placed on the same side as a maturing AV access if possible. Femoral catheters should be placed only in bedbound patients only with good exit site care and should be, left in place for no more than 5 days because of associated infection rates. Reference: APIC"Text, 4th edition, Chapter 39

29. The IP is reviewing the facility's performance measures, which are used to benchmark against national data. The IP ensures that each performance measure includes which of the following characteristics: 1) Measure is reliable 2) Measure targets improvement in a health population 3) Measure is defined according to physician preference 4) Measure can be easily interpreted by the users of the data a. 1, 2, 3 b. 1, 2, 4 c. 2, 3, 4 d. 1, 3, 4

1, 2, 4 Rationale: Performance measures focus on outcomes or processes. They are used for internal improvement purposes, intra- or inter-organizational comparisons, and by various external entities for making decisions about care. Performance measure should be designed to address improvement that is likely to have a significant impact to the health of a specified population. The measure should consistently track the events within an organization or across organizations and over time. The resulting data should be easily understood by the end-users (e.g., staff, facility leaders). Reference: Chapter 17

69. Inadequate refrigeration of food may permit the growth of potentially harmful microbes. Interventions to prevent the growth of pathogens due to inadequate refrigeration include: 1) Train personnel to recognize and implement safe maintenance of refrigerators 2) Establish a method to record temperature on a regular basis 3) Make daily rounds to ensure that the freezer and refrigerator are clean 4) Schedule and perform regular preventive maintenance of freezer and refrigerator a. 1, 2, 3 b. 1, 2, 4 c. 2, 3, 4 d. 1, 3, 4

1, 2,4 Rationale: Interventions to prevent the growth of microbes include: • When selecting/purchasing equipment for cooling or freezing, compare features that best meet the intended use, including operating range (e.g., an automatic defrost cycle can damage temperature-sensitive items), size, location of use, cleanable surfaces, durability, and maintenance needs. . • Provide accurate temperature monitoring for refrigerators and freezers; an alarm system may be required (e.g., blood bank refrigerator) or desired. • Establish a method to record temperature on a regular basis (e.g., visualize and document daily or observe an automated recording chart each shift); include action to take if reading is not in the acceptable range. • Schedule routine 'monitoring of refrigerator and freezer alarms where applicable. • Test accuracy of thermometers; calibration may be required (e.g., blood bank, tissue freezer) by. using standard regulations and/or recommendations. • Schedule and perform regular preventive maintenance of all freezers and refrigerators; include air vents, gaskets, cooling coils, and fans. • Walk-in refrigeration units may experience a condensation point if the building dehumidification is inadequate, resulting in mold proliferation. • Provide training for personnel in reciognizing and implementing safe maintenance of refrigerators to include appropriate cleaning methods. Reference: APIC Text; 4th edition, Chapter 112

3. Which of the following rules should be followed when collecting a stool sample for C. difficile testing? 1) Stool sample should be freshly passed within 1-2 hours 2) 10-20rnL. of formed stool should be collected 3) Stool should be passed into a clean, dry container 4) Specimens should be obtained before antimicrobial agents have been administered a. 1,2 b. 2,3 c. 1,3 d. 1,4

1, 3 1) Stool sample should be freshly passed within 1-2 hours 3) Stool should be passed into a clean, dry container Rationale: The accuracy of all tests depends on proper specimen handling and transport. The following rules should be followed when collecting samples for C. difficile testing: • Stool samples should be freshly passed within 1-2 hours • 10-20 mL of watery, soft, or unformed stool should be collected • Stool should be passed into a clean, dry container Reference: APIC Text, 4th edition, Chapter 72

75. A case of healthcare-associated Mycobacterium chelonae respiratory infection has been identified in a patient. Of the following exposures that the patient had in the facility, which should be investigated as a potential source of the infection? 1) Bronchoscopes 2) Dialysis 3) Hydrotherapy pool 4) Ice from tap water used during surgery a. 1 b. 1, 3 c. 1, 3, 4 d. 1, 2, 3, 4

1, 3 Rationale: Mycobacterium chelonae belongs to the family of nontuberculous mycobacteria (NTM) classified in the rapidly growing mycobacteria (RGM), Runyon group IV. M chelonae are ubiquitous in the environment and have been isolated from both natural and potable freshwater sources, soil, contaminated solutions, and reptiles. The organism can grow in distilled and unsupplemented water. Likely sources of healthcare-related exposure may occur from bronchoscopes that were processed with tap water or from exposure to a hydrotherapy pool. Reference: CDC Guidelines for Environmental Infection

86. Important elements of surveillance programs include: 1) Tracking diseases associated with the healthcare environment 2) Environmental sampling 3) Reviewing microbiology reports for antibiotic resistant organisms • 4) Tracking and reporting HAls as mandated by state/local public health requirements -

1, 3, 4 Rationale: Surveillance has been defined as ,the "ongoing collection, collation, and analysis of data and the ongoing dissemination of information to those who need to know so that action can be taken." It is an essential component of an effective infection prevention program. Surveillance programs should measure outcomes of healthcare, processes of healthcare, and selected events of importance to the organization. Routine or random, undirected microbiological culturing of air, water, and environmental surfaces in healthcare facilities is not recommended. . Reference: APIC Text, 4th edition, Chapter 11

63. An early-level (novice) IP in your department has set a goal of advancing to achieve middle-level (proficient) competency within the next year. Which of the following activities would be the most appropriate to include on her personal development plan for the year: 1) Nominating herself for the President-elect position of the local APIC chapter 2) 'Taking the Certification in Infection Control exam within six months 3) Requesting information about a Master of Science in Epidemiology degree 4) Learning the basics of CAUTI surveillance a. 1 b. 1,2 c. 1, 2, 3 d. 1, 2, 3, 4

1,2,3 Rationale: APIC has created a competency model to help guide the advancement of infection preventionists in the field. The three levels of competency are early-level (novice), middle-level (proficient), and advanced level (expert). The competency levels can be used to guide goal setting activities as part of the IP's personal development plan. Middle-level competencies include being Certified in Infection Control, considering a`h advanced degree in the field, and being active in the local APIC chapter by serving in a -leadership position. Reference: APIC Text, 4th edition, Chapter 2

62. Program evaluation is necessary to measure change and growth in the learner. The following program elements should be evaluated in order to demonstrate efficacy and impact. 1) Appropriateness of the program design 2) Adequacy of the teaching and instructional resources 3) Knowledge, skills, and attitudes learned by the pariicipants 4) Assessing the educational needs of the attendees a. 1, 2, 3 b. 1, 3, 4 c. 2, 3, 4 d. 1, 2, 4

1,2,3 Rationale: Program evaluation is a systematic method for collecting, analyzing, and using information to assess the effectiveness and efficiency of the educational offering. Specific program elements that must be evaluated include appropriateness of program design, adequacy of teaching and instructional resources, and the knowledge, skills, and attitudes learned by the participants. Needs assessments identify deficiencies in knowledge, skills, or attitude and should be conducted prior to the development of the program. Reference: APIC Text, 4th edition, Chapter 3

41. Important considerations regarding blood culture specimens include: 1) Collect prior to the initiation of antimicrobial therapy 2) Collect from a central venous catheter whenever possible 3) Ensure that the volume of the specimen collected is sufficient 4) Culture of specific sites is not recommended for surveillance a. 1,3 b. 2, 4 c. 1,4 d. 3,4

1,3 Rationale: The accuracy of a blood culture can be impacted by a wide variety of factors, many of which pertain to skin antisepsis and/or specimen collection techniques. The venipuncture site should be cleaned with an antiseptic first to minimize the risk of contaminating the blood specimen with common commensals. It is critical that blood cultUres be drawn prior to initiation of antibiotic therapy. Blood may not be sterile immediately following antimicrobial therapy. If empiric antibiotic therapy is initiated on an emergency basis, cultures should be obtained as soon as possible following the first dose. The volume of blood obtained for culture is a critical variable in detecting bacteremia or fungemia. Specimen collection from a central venous catheter is not recommended due to the risk of intraluminal bacterial contamination of the device. Percutaneous venipuncture from two separate sites is preferred. References: APIC Text, 4th edition, Chapter 24

9. A patient who underwent intraocular surgery was diagnosed with a noninfectious endopthalmitis after the procedure. The IP initiates an investigation to identify the possible cause. Which of the following factors should be considered? 1) Improper handling, cleaning, and rinsing of the instruments 2) Improper labeling of the solutions 3) Gloves and powder 4) Prophylactic antibiotics administered 2 hours before the procedure a. 1,3 b. 1,2 c. 3,4 d. 2,4

1,3 1) Improper handling, cleaning, and rinsing of the instruments 3) Gloves and powder Rationale: Endophthalmitis is an inflammatory condition of the intraocular cavities (aqueous and/or vitreous humor) usually caused by infection. Noninfectious (sterile) endophthalmitis may result from various causes such as retained native lens material after an operation or from toxic agents. Improper cleaning and rinsing of surgical instruments can leave a residue, which can irritate the eye and cause an inflammation. Gloves, especially those with powder, can also cause inflammation of the eye during surgery. Only a few organisms can penetrate the intact epithelium of the conjunctiva or cornea. Among these are Neisseria gonorrhoeae, Neisseria meningitidis, Streptococcus pneumoniae, Listeria monocytogenes, and Corynebacterium diphtheriae. For all others, a breach in the protective epithelial barrier or mucous membranes must occur. Once the intact epithelium has been breached (e.g., by trauma, or by insertion/removal of a contact lens), any microorganism gaining entrance can cause disease Reference: APIC Text, 4th edition, Chapter 63 - Ophthalmology Services

76. Epidemic kerato-conjunctivitis (EKC) is a viral conjunctivitis caused by a group of adenoviruses. EKC is highly contagious and can be problematic in ambulatory surgery settings. Recommended measures for control of these infections include: 1) Disinfection of tonometer tips in 3 percent hydrogen peroxide 2) Sterilize ophthalmoscopes between uses 3) Disinfection of the environment 4) Frequent hand hygiene a. 2, 3, 4 b. 1, 2, 4 c. 1, 3, 4 d. 2, 3, 4

1,3,4 Rationale: Prevention of EKC requires meticulous attention to hand washing. Soap and water and/or an alcohol-based hand sanitizer should be used before and after each patient contact. Gloves should be worn and discarded appropriately during outbreaks and when exposure to patient's tears or excretions is likely. The current CDC recommendations for disinfection of tonometer tips include a 5 to 10 minute soak in 3 percent hydrogen peroxide, 70 percent isopropyl, 70 percent ethyl alcohol, or in 5,000 ppm bleach. Ophthalmoscopes should be wiped with 70 percent alcohol between patients. Reference: APIC Text, 4th edition, Chapter 63

100. A patient has been admitted with a wound infection. The lab reports that the stain of the wound culture is positive for AFB and the culture is positive for Mycobacterium marinum. Which of the following is the correct follow-up of this lab report? a. No further follow-up is needed b. Place the patient on airborne isolation c. Contact the local health department to report the Mycobacterium infection d. Place the patient on Contact Precautions

100. 0 No further follow-up is needed Rationale: M. marinum is a bacterium found in water and may be the cause of wound infections. Infection with M. marinum is not transmissible, and infection with non-tuberculosis mycobacteria is not notifiable. No further infection control follow-up is needed.

101. Examples of efforts to improve patient safety in the healthcare setting include: 1) Encouraging patients to ask their healthcare providers if they have washed their hands 2) Medication safety programs that report medication errors and inform staff of efforts to prevent repeated errors 3) Encouraging patients to ask questions about their treatments and medications 4) Emphasizing punitive reactions to mistakes a. 1, 2, 4 b. 1, 2, 3 c. 2, 3, 4 d. 1, 3, 4

101. 0 1, 2, 3 Rationale: In healthcare organizations, surveillance, reporting, and analysis are the foundation of risk prevention programs, but targeted interventions must be deployed if patient safety programs are to be successful in reducing harm from medical errors and other adverse events. The Agency for Healthcare Research and Quality recommends that all healthcare organizations focus on the following infection prevention initiatives: 1. Improving hand hygiene 2. Utilizing barrier precautions to prevent transmission of infection 3. Prudent antibiotic use to reduce C. difficile and VRE 4. Preventing urinary tract infections S. Preventing central venous catheter-related bloodstream infections 6. Preventing VAP 7. Preventing SSIs Medication errors are also common in healthcare and are addressed in TJC's list of sentinel events: "Any patient death, paralysis, coma, or other major permanent loss of function associated with a medication error." A culture of safety must prevent punitive reactions to mistakes, and staff members must feel confident that if they speak out about risk, their leaders will respond. Providers involved in medical errors must know that leaders will look beyond the obvious and drill down until the root causes of accidents and errors are found and that they will routinely evaluate systems and processes during any accident investigation. Reference: APIC Text, 4th edition, Chapter 18

102. Which short-term central lines are associated with a higher risk of infection? 1) Peripherally-inserted central venous catheters (PICC) 2) Femoral lines 3) Subclavian lines 4) Jugular lines a. 1,2 b. 2,3 c. 2,4 d. 1,4

102. cip 2, 4 Rationale: The CDC recommends using the subclavian site rather than a jugular or a femoral site in adult patients to minimize infection risk for nontunneled central venous catheter placement.

103. A technician finds out after obtaining an EKG on a patient that the patient may have varicella-zoster (shingles) on a dermatome on the upper body. The Occupational Health Nurse checks the employee's records and realizes that the employee was never tested for varicella on hire. The first thing the Occupational Health Nurse should do is: a. Determine if the patient actually has an active case of varicella-zoster by involving the IP or checking with the patient's physician to verify the diagnosis b. Test the employee for varicella immunity and, if not immune, exclude from work from day 10 through day 21 after the exposure c. Give the varicella vaccine to the employee d. Give varicella-zoster immune globulin (VZIG) to the employee

103. 0 Determine if the patient actually has an active case of varicella zoster by involving the IP or checking with the patient's physician to verify the diagnosis Rationale: The first step is to verify that the patient actually has the disease. If the patient does live confirmed varicella-zoster, the occupational health nurse should work this up as an exposure and exclude the employee from work. References: APIC Text, 4th 'edition, Chapter 100

104. A healthcare worker has had her uniform scrub top contaminated by a large blood spill that has penetrated the fabric. Choose her best option for follow-up from the choices below. a. She should be sent home immediately to take a shower b. She should remove the top by carefully pulling itover her head and then launder it in a washing machine with hot water and bleach c. She should remove the scrub top by using scissors to cut it off and discard it in a biohazard-labeled container d. She should remove the top by pulling it over her head and then discard it in the trash

104. e She should remove the scrub top by using scissors to cut it off and discard it in a biohazard-labeled container Rationale: A scrub top that has been thoroughly soiled with blood should be removed immediately. If the scrub top is soiled through the top layer, there is potential for further exposure through contamination of mucus membranes if the employee pulls it over her head. She should use scissors to carefully cut off the top. Reference: APIC Text, 4th edition, Chapter 111

105. There was contamination of a healthcare facility's potable water system with sewer water and there is high concern for the presence of Cryptosporidium. Which of the following should you recommend for corrective decontamination of the system? a. Use chlorination to disinfect the system b. Use high-temperature flushing to disinfect the system c. Use fluoridation to flush the system d. Use ultraviolet light to flush the system

105. 0 Use high-temperature flushing to disinfect the system Rationale: If there is contamination of the potable water system with sewer water in a facility then it is necessary to flush the system. A variety of methods can be used to accomplish this and they may be used in combination, if necessary. As Cryptosporidium is highly resistant to chlorine and there is suspected contamination with this protozoa in this scenario,. you would have to minimally recommend the use of high-temperature flushing to disinfect the system.

106. During environmental rounds, the IP observes an environmental service worker cleaning up a blood spill on the floor. The best choice for cleaning blood and body fluids is: a. Alcohol b. Quaternary ammonium compound c. Phenolic d. Bleach

106e. B leach Rationale: Many disinfectants are inactivated by blood and other organic matter. If the spill is small, it can be cleaned and disinfected in one step by pouring the disinfectant directly on the spill and cleaning up after the appropriate contact time for the disinfectant has been reached. If the spill involves a higher concentration of microorganisms, such as a large body fluid spill or a blood spill in a laboratory, absorbent material can be placed over the spill until all of the fluid is absorbed. The disinfectant solution can be poured over the spill and the absorbent material until both are thoroughly soaked with the disinfectant for the designed contact time. Then the spill site is cleaned. Absorbent powders that solidify liquids are recommended for cleaning large spills. Occupational Safety & Health Administration (OSHA) stipulates that blood and body fluid spills be decontaminated with one of the following: an EPA registered disinfectant effective against HBV and HIV, a tuberculocidal disinfectant, or an appropriate dilution of household bleach (5.25 to 6.15 percent sodium hypochlorite solution). If the spill occurs on a nonporous surface, a 1:100 dilution of household bleach (one part household bleach plus 99 parts tap water or 0.25 cup of bleach in a gallon of water) is a highly effective disinfectant and is the least expensive. A 1:10 dilution of household bleach (one part household bleach plus nine parts water or 1.5 cups of bleach in a gallon of water) is required for porous surfaces and large spills. Large quantities of blood may inactivate the higher concentration of bleach. Higher concentrations of bleach may also be damaging or corrosive to surfaces. Reference: APIC Text, 4th edition, Chapter 107

108. Which of the following quality concepts will identify a pattern of observation points above and below the mean level? a. Fishbone diagram b. Run chart c. Pareto chart d. Process flow chart

108. Q Run chart Rationale: Run charts are epidemiological tools used to identify how process specifications change over time. For example, if the organization examines the number of HAls occurring within a hospital over a given period of time, clinicians will record each monthly value and graph the values, creating a chart that looks at trends and averages. Run charts allow for the mean or average to be determined and show changes in the mean/average. Run charts also demonstrate special cause variation when there is a steady pattern of observation points falling above or below the mean/average line in an equal pattern. Reference: APIC Text, 4th edition, Chapter 16

80. All of the 72 patients in a chronic hemodialysis center were tested for Hepatitis C virus (HCV). Eight of the patients were identified as HCV positive. What is the prevalence? a. 0.9 percent b. 9 percent C. 1 percent d. 11 percent

11 percent Rationale: A prevalence rate is the proportion of persons in a population with a particular disease or attribute at a specific point in time (point prevalence) or over a specified time period (period prevalence). Prevalence depends on the duration of disease. The prevalence rate equals the number of existing cases of disease from a specified interval or point in time divided by the population at risk for same time period multiplied by a constant (k). k = A constant used to transform the result of division into a uniform quantity so that it can be compared with other, similar quantities. A whole number (fractions are inconvenient) such as 100, 1,000, 10,000, or 100,000 is usually used (selection of k is usually made so that the smallest rate calculated has at least one digit to the left of the decimal point) or is determined by accepted practice (the magnitude of numerator compared with denominator). This prevalence rate is calculated as: 8 ÷ 72 x 100 = 11 percent Reference: APIC Text, 4th edition, Chapter 13

111. An outbreak of Hepatitis C has occurred in the endoscopy clinic. Single-dose vials of medication are used in the clinic and observation of practice shows that the rubber stopper of each vial is wiped with alcohol before access. Medication is drawn up with a needle and syringe and administered to the patient. If an added dose is needed, the needle is replaced on the syringe and more medication is drawn up from the vial. If medication remains in the vial, then it is stored at the recommended temperature for use on another patient. HCP in the clinic had a 40 percent overall compliance with recommended hand hygiene practices. Which practice is the most likely cause of the outbreak? a. Use of a single-dose mediation vial for more than one patient b. Poor hand hygiene practices c. Contamination of medication through the rubber stopper d. Inadequate cleaning and disinfection of endoscopes

111. 0 Use of a single-dose medication vial for more than one patient Rationale: Vials that are labeled as single-dose or single-use should be used for a single patient and single case /procedure/injection. There have been multiple outbreaks resulting from healthcare personnel using single dose or single-use vials for multiple patients. Even if a single-dose or single-use vial appears to contain multiple doses or contains more medication than is needed for a single patient, that vial should not be used for more than one patient nor stored for future use on the same patient. Reference: APIC Text, 4th edition, Chapter 110

113. The IP wants to calculate the surgeon-specific rate of infections associated with spinal fusion surgeries in the facility. Which of the following data elements will he need? 1) Number of spinal fusion procedures within the designated time frame 2) Number of spinal fusion SSIs within the designated time frame 3) Number of spinal fusion surgeries performed by each surgeon within the designated time frame 4) Number of spinal fusion infections for each surgeon within the designated time frame a. 1,2 b. 1, 4 c. 2,3 d. 3, 4

113e. 3, 4 Rationale: The IP will need to divide the number of spinal fusion infections for each surgeon by the number of spinal fusion surgeries performed by each surgeon. Reference: APIC Text, 4th edition, Chapter 21

114. A blood spill contaminated a pneumatic tube system and its contents. The leak was not identified for a period of time. After remediation of the contamination, the IP recommended the following to prevent a future incident: a. Establish a method to recognize spills or leaks b. Perform a hazard vulnerability analysis c. Contract with an outside vendor to clean and maintain system d. Restrict the use of pneumatic tube to nonhazardous material

114. 0 Establish a method to recognize spills or leaks Rationale: Blood/body fluid (e.g., from specimens due to spill or leak) may contaminate transport system pathways (e.g., pneumatic tube system), receiver stations, transport carts, dumbwaiters, or elevators. Disruption caused by these spills can delay specimen transport and processing until the area is cleaned and disinfected. Personnel may also be exposed to bloodborne pathogens, other microbes (e.g., sputum specimen, tissue specimen), or chemicals (pharmaceuticals; preservatives such as formaldehyde). Establish a method to recognize spills or leaks; include communication protocol (e.g., notification to sender, system shutdown, alternative methods of transport), cleaning and decontamination, retrieval of "stuck" items, and restoration of system. Reference: APIC Text, 4th edition, Chapter 112

115. A healthcare facility is undergoing extensive renovation. Surveillance for which of the following organisms would be particularly important during any construction or renovation project in a healthcare facility? a. MRSA, VRE, and other MDROs b. Leg/one/la and Aspergillus c. Gram-negative bacilli d. Mycobacterium abscessus

115. 0 Legionella and Aspergillus Rationale: The risk of HAls increases significantly when hospitals are undergoing construction and renovation. Dust particles contaminated with bacteria and fungi are dispersed and pose a health risk for patients, staff, and visitors. Construction-related HAls are primarily caused by fungi, and to a lesser extent by bacteria. The most common etiological agent is Aspergillus, in particular, A. fumigatus, A. flavus, A. niger, and A. terreus. A. fumigatus is considered the most pathogenic species and is responsible for more than 90 percent of all Aspergillus infections. The most common group of bacteria associated with construction-related nosocomial infections are Leg/one/la species, including L. pneumophila. Legionella bacteria have been isolated from soil and dust but are more often associated with aquatic environments. Typical water sources in hospitals supporting colonization with Legionella bacteria are cooling towers, evaporative condensers, heated potable water systems, and heating and air conditioning systems. During construction and renovation processes, water systems are often disrupted. Potable water can become contaminated with Leg/one/la when the water supply is restored. The introduction of contaminated soil into the plumbing system, as well as optimized growth conditions for bacteria in stagnant water, may increase the number of Legionella bacteria. References: APIC Text, 4th edition, Chapter 11

116. An outbreak of norovirus in an LTCF would most likely have an epidemiological curve (EPI curve) that: a. Indicated a common source of infection b. Indicated .a propagated source of infection c. Indicated a point source of infection d. Indicated a common vehicle of infection

116. 0 Indicated a propagated source of infection Rationale: An epidemic curve gives a graphical display of the numbers of incident cases in an outbreak or epidemic plotted over time. The form of the resulting distribution of cases can be used to propose hypotheses on the nature of the disease and its mode of transmission. A propagated source means that infections are transmitted from person to person in such a way that cases identified cannot be attributed to agent(s) transmitted from a single source. Propagated (continuing) source cases occur over a longer period than in common source transmission. Explosive epidemics resulting from person-to-person transmission may occur (e.g., chickenpox). If secondary and tertiary cases occur, intervals between peaks usually approximate average incubation period. Reference: APIC Text, 4th edition, Chapter 12

117. What is the term for an unexpected occurrence involving death or serious physical or psychological injury, or the risk there of? a. Error b. Adverse event c. Near miss d. Sentinel event

117. Q Sentinel event Rationale: According to The Joint Commission, a sentinel event is an unexpected occurrence involving death or serious physical or psychological injury or the risk thereof. Serious injury specifically includes loss of limb or function. The phrase "or the risk thereof" includes any process variation for which a recurrence would cacry a significant chance of a serious adverse outcome. Such events are called sentinel because they signal the need for immediate investigation and response. Reference: APIC Text, 4th edition, Chapter 18

119. A patient in has been diagnosed with Legionnaires' disease that was possibly acquired during his stay in the hospital. What is the recommended firs t response to this incident? a. All water sites in the healthcare facility to which the patient was exposed should be cultured for L. pneumophlla b. The patient should be placed on respiratory isolation until 24 hours after treatment has started c. The water system should be superheated and flushed immediately d. All patients should immediately be provided with bottled water for drinking

119. 0 All water sites in the healthcare facility to which the patient was exposed should be cultured for L. pneumot•hila Rationale: If there is suspicion of healthcare-associated L. pneumophila, the water system should be tested first to determine whether the microbe is present. References: APIC Text, 4th edition, Chapter 84

110. You are the IP in an ambulatory surgery center. During rounds, you discover that staff are using a single bag of saline to prepare saline flushes for multiple patients. Your immediate response is: a. Clarify that saline bags must be labeled with time of use and discarded within 24 hours b. Explain that single dose bottles of sterile water should be used to flush IVs c. -Remove the bag and send a follow-up email to the staff explain your actions d. Remove the bag and teach staff how to follow safe injection practices

11e0. R emove bag and teach staff how to follow safe injection practices Rationale: Unsafe injection practices put patients and healthcare providers at risk of infectious and non-infectious adverse events and have been associated with a wide variety of procedures and settings. Use fluid infusion and administration sets for one patient only and dispose of appropriately after use. Do not use bags or bottles of IV solution as a common source of supply for multiple patients. References: APIC Text, 4th edition, Chapter 64

120. Properly written instructional objectives include: a. Statements that communicate the intent of the curriculum b. Directions and plans for the educational session c. Learner outcomes in measurable terms using action verbs d. Measures of change and growth in the learner

120. G Learner outcomes in measurable terms using action verbs Rationale: Instructional objectives include specific actions the learner will perform as a result of instruction. There is no single correct method or style, but properly written learning objectives describe the learner outcomes in measureable terms and use action verbs such as discuss, describe, or demonstrate. Statements that communicate the intent of the curriculum and directions and plans for the educational session (A and B) are more appropriate for learning goals. Measures of changes, and growth in the learner (D) are used for evaluation of the learning program. Reference: APIC Text, 4th edition, Chapter 3

121. The key to cleaning and disinfection of environmental , surfaces is the: a. Use of specific antiseptics for each surface b. Physical removal of visible dirt, Organic material, and debris c. Assignment of trained staff to each department d. Maintenance of equipment used for cleaning

121. 0 Physical removal of visible dirt, organic material, and debris " Rationale: Accumulation of dust, soil, and microbial contaminants on environmental surfaces in healthcare facilities is not only aesthetically displeasing, but evidence indicates that it also plays a major role in the spread of HAls. Studies have shown that many microorganisms survive on surfaces for long periods. The key to cleaning and disinfecting environmental surfaces is the use of friction to physically remove visible dirt, organic material', and debris, thereby removing microorganisms.

129. A patient who had neurosurgery was exposed to Creutzfeldt- Jakob disease (CJD) prions through surgical instruments that were not correctly processed after having been previously used on an infected patient. As a result, she has developed the disease herself. This is an example of which mode of infection? a. Familial CJD b. latrogenic CJD c. Spontaneous CJD d. Mutational CJD

129. 0 latrogenic CJD Rationale: CJD is one of several neurologically degenerative diseases caused by a group of protein particles that are infectious by nature of their ability to replicate in the central nervous system and interrupt crucial neuron functioning. CJD and other prion diseases with demonstrated transmissibility remain a concern for the healthcare community because of their inherent resistance to traditional disinfection/sterilization methods and devastating clinical outcomes. Prion diseases occur sporadically in nature, by familial transmission (gene mutation), iatrogenically, and by ingestion of abnormal prions as in the case of the bovine encephalopathies. In this instance, the CJD developed as a result of medical treatment so it is termed iatrogenic. Reference: APIC Text, 4th edition, Chapter 73

130. List the following diseases in order of resistance of the disease agents to disinfection and sterilization, from high resistance to low resistance. 1) Staphylococcus aureus 2) Clostridium difficile 3) Creutzfeldt-Jakob Disease 4) Polio a. 1, 2, 3, 4 b. 2, 3, 4, 1 c. 3, 4, 2, 1 d. 3, 2, 4, 1

130. e 3, 2, 4, 1 Rationale: Microbes and other disease causing agents such as prions have . differing resistance to disinfection and sterilization. Prions are the most resistant and special reprocessing is required for instruments that have come in contact with infectious tissue of a person with Creutzfeldt-Jakob Disease. Spore-forming microbes are less resistant than prions but sterilization is require to remove these agents from medical instruments. Non lipid viruses such as Poliovirus and small viruses require intermediate-level disinfection and are more resistant than lipid-viruses or larger viruses. Vegetative bacteria such as Staphylococcus aureus have the lowest level of resistance to disinfection and sterilization.

132. What is the positive predictive value of the following data? a. 97.0 percent b. 92.5 percent c. 96.2 percent d. 99.4 percent

132. 0 97.1 percent Rationale: The positive and negative predictive values are the proportions of positive and negative results in statistics and diagnostic tests that are true positive and true negative results. The positive predictive value is the proportion of people with positive test results who have the disease. Positive test True positive,(TP) False positive (FP) Negative test Fels negative (FN) True negative (TN) Positive predictive value = TPATP + FP) The positive predictive value for this question is calculated as follows: 100 ÷ (100 + 3) = 0.97 or 97 percent Reference: APIC Text, 4th edition, Chapter 13

133. All of the following would be considered prevention strategies to reduce the transmission of infections in a healthcare setting except: a. Implementing an influenza vaccine program for employees b. Educating staff members in PPE usage c. Decolonization of MRSA-positive patients prior to surgery d. Instituting use of bleach for cleaning C. difficile patient rooms

133. Decolonization of MRSA-positive patients prior to surgery Rationale: Prevention strategies in healthcare infection prevention are wide ranging and depend on the disease in question and what information is available to the practitioner. Prevention strategies to reduce the risk of transmission, including barrier precautions; immunizations of HCP; and cleaning, sterilization, and disinfection are designed to prevent the occurrence of disease and, therefore, form primary prevention, measures. Current decolonization-protocols using intranasal mupirocin and chlorhexidine body washes are effective for reducing MRSA colonization in surgical patients. The purpose of this treatment is to prevent SSIs in the individual patients. It is not designed to prevent the spread of MRSA to other patients. Reference: APIC Text, 4th edition, Chapter 10

118. The incubation period for meningococcal meningitis is: a. 10 to 14 days b. 7 to 21 days c. 2 to 10 days d. 1 to 2 days

2 to 10 days Rationale: The incubation period for meningococcal meningitis is between 2 to 10 days, commonly 3 to 4 days.

131. When considering occupational health issues in healthcare settings, which individuals are covered under the term "healthcare personnel"? 1) All paid persons working in healthcare settings who have the potential for exposure to infectious materials 2) All paid and unpaid persons working in healthcare settings who have the potential for exposure to infectious Materials 3) Any individual who has the potential to acquire or transmit infectious agents during the course of his or her work in healthcare 4) All paid and unpaid persons who-work in healthcare settings and encounter patients 5) All workers employed by the healthcare organization a. 1,5 b. 2,3 c. 1,4 d. 4, 5

2, 3 Rationale: According to the CDC, paid and unpaid personnel, as well as contracted workers or workers employed outside the healthcare organization (such as emergency medical service personnel) are included in the term HCP. Also included are laboratory and autopsy personnel as well as researchers and volunteers who may potentially be exposed to infectious agents. Reference: APIC Text, 4th edition, Chapter 100

77. All of the 72 patients in a chronic hemodialysis center were tested for Hepatitis C virus (HCV). Eight of the patients were identified as HCV positive. During the following year, two of the dialysis center's patients who previously tested negative for HCV converted to HCV positive. What was the incidence for that year? a. 2.8 percent b. 3.1 percent c. 13.8 percent d. 7.2 percent

3.1 percent Incidence rate in this scenario: 2 ÷ 64 x 100 = 3.125 percent Rationale: An incidence rate is a measure of the frequency with which an event occurs in a population over a specified period of time. Incidence indicates the risk of disease in a population over a period of time. The incidence rate equals the number of new cases of a disease for a specified time period divided by the population at risk for the same time period multiplied by a constant (k). k = A constant used to transform the result of division into a uniform quantity so that it can be compared with other, similar quantities. A whole number (fractions are inconvenient) such as 100, 1,000, 10,000, or 100,000 is usually used (selection of k is usually Made so that the smallest rate calculated has at least one digit to the left of the decimal point) or is determined by accepted practice (the magnitude of numerator compared with denominator). The numerator for this scenario is 2—the number of new cases. The denominator would be the number of patients who are at risk for developing HCV, which would be 64. Patients already diagnosed with HCV would not be included. Reference APIC Text, 4th edition; Chapter 13

125. In 2013, 565 persons died from influenza-related illness in a large metropolitan area with a population of 1.8 million. What was the cause-specific mortality rate? a. 31 per 100,000 b. 53 per 100,000 c. 31 percent d. 0.03 percent

31 per 100,000 565 ÷ 1, 800,000 x 100,000 = 31.38 Rationale: A mortality rate is the measure of the frequency of death in a defined population during a specified time (usually a year). The crude mortality rate measures the proportion of the population dying each year from all causes. The cause-specific mortality rate measures mortality from a specified cause for a population. Mortality rate = x/y x k Where: x = The number of people in a defined population during a specified interval of time who (1) die of any cause (crude rate) or (2) die of a specified cause (cause-specific rate) y = Estimated population at midyear (i.e., July); crude rates use 1000 or 100,000 k = Usually an assigned value of 1,000 when calculating crude rates: 100,000 is used for cause-specific rates The cause-specific mortality rate for this scenario is calculated as follows: 565 ÷ 1, 800,000 x 100,000 = 31.38 We round to 31 and state that the cause-specific mortality rate is 31 per 100,000. Reference: APIC Text, 4th edition, Chapter 13

46. Which of the following is not an infection prevention objective of an occupational health program? a. Contain costs by preventing infectious diseases that result in absenteeism and disability b. Provide care to personnel for work-related illnesses or exposures c. Educate patients about the principles of infection prevention d. Collaborate with the Infection Prevention Department in monitoring and investigating potentially harmful infectious exposures and outbreaks

46. 43 Educate patients about the principles of infection prevention Rationale: The healthcare organization's administration, medical staff, and other HCP need to support the infection prevention objectives of an occupational health program. These objectives are to (1) educate personnel about the principles of infection prevention and their individual responsibility for infection prevention, (2) collaborate with the Infection Prevention Department in monitoring and investigating potentially harmful infectious exposures and outbreaks, (3) provide care to personnel for work-related illnesses or exposures, (4) identify work-related infection risks and institute appropriate preventive measures, and (5) contain costs by preventing infectious diseases that result in absenteeism and disability. Reference: APIC Text, 4th edition, Chapter 100

48. What action is indicated when the IP is asked to help determine if a worker has experienced occupational acquisition of an infectious agent or disease in order to receive workers' compensation benefit? a. Provide enough information to prove or disprove the employee's claim b. Notify the facility's attorney immediately c. Review the workers' compensation system in place d. Perform a root cause analysis to investigate

48. Q Review the workers' compensation system in place Rationale: Disease that results from occupational exposure usually is eligible for compensation if the occupational exposure is the sole cause of disease; the occupational exposure is one of several causes of the disease; the occupational exposure aggravates a pre-existing disease (e.g., asthma); or the occupational exposure hastens the onset of disability. The burden of proving that disease was occupationally acquired lies with the workers. The IP's expertise may be needed to help assess this situation. Risk Management and Occupational Health provide the necessary follow-up. IPs should be familiar with the workers' compensation system in place within their country. Reference: APIC Text, 4th edition, Chapter 100

49. The safe temperature range for cold food storage is: a. 41°F/5°C or lower b. 42°F to 50°F/5.6°C to 10°C c. 50°F to 55°F/10°C to 12.8°C d. Less than 60°F/15.6°C

49. ID 41°F/5°C or lower Rationale: Cold foods for serving must be held at 41°F/5°C:or lower. The temperature danger zone is 41°F to 135°F/5°C to 57°C, which is the temperature range for rapid multiplication of virtually all bacteria associated with foodborne disease. Reference: APIC Text, 4th edition, Chapter 109 - Nutrition

50. A robust Performance Improvement team should perform all of the following except: a. Observational audits b. Benchmark comparisons c. Root cause analyses d. Housewide infection rates

50. Q Housewide infection rates Rationale: A robust performance improvement program should include the following basic elements: measuring how a facility or organization controls or complies with policies; documenting results of observational audits; performing root cause analyses; reporting individual physician or unit infection rates; and benchmarking the organization's infection rate against community, state, and national averages.. Reference: APIC Text, 4th edition, Chapter 16

51. Toys used for inpatient pediatric patients should ideally be all of the following except: a. New b. Nonporous c. Plush d. Single-patient use

51. G Plush Rationale: Strategies to minimize pediatric infection risk from toys include: • Practice of hand hygiene by patients before and after handling toys • Implementation of a process for appropriate toy acquisition to ensure suitability for cleaning/disinfection; toys should be nonporous and able to withstand rigorous mechanical cleaning. Avoid toys that are waterretaining, plush, and stuffed (an exception is therapeutic dolls, which should be single-patient use); and those that are difficult to clean and dry. • Use of new toys Reference: APIC Text, 4th edition, Chapter 42

52. Which of the following would be an appropriate method to evaluate the quality of an infection prevention program? a. The total number of areas where surveillance was carried out in the past year b. The average amount of time that elapsed between receiving reports from the lab about patients with multidrug-resistant infections and placing those patient on appropriate Isolation Precautions c. The number of IPs in the program per the number of beds d. The average amount of money spent on isolation gowns this year as compared to last year

52. 0 The average amount of time that elapsed between receiving reports from the lab about patients with multidrug-resistant infections and placing those patients on appropriate Isolation Precautions Rationale: The quality of the infection prevention program should be assessed routinely by evaluating customer satisfaction, appropriateness, efficacy, timeliness, availability, effectiveness, and efficiency. The average amount of time that elapsed between receiving reports from the lab about patients with multidrug-resistant infections and placing those patients on appropriate Isolation Precautions relates to timeliness of initiating appropriate interventions. Therefore, it can be used as a quality measure for the program. Reference: APIC Text, 4th edition, Chapter 1

54. An employee is exposed to blood and body fluids from a patient whose baseline testing revealed positive results in a rapid HIV test. The most appropriate follow-up test for the patient would be: a. Western Blot b. Viral Load c. HIV polymerase chain reaction d. CD4

54. 0 Western blot Rationale: Rapid HIV tests can offer a result in 15 minutes. A negative rapid test is reported as a definitive negative HIV test. A reactive test result needs to be confirmed with standard serologic tests. The most commonly used laboratory test for diagnosis of HIV infection is the serologic detection of antibodies to the virus. The standard serologic test consists of a screening enzyme immunoabsorbent assay (EIA) followed by a confirmatory Western blot (WB). In a patient with a positive EIA test, the test should be repeated. In a patient with a repeatedly positive EIA, test, a confirmatory WB is performed. These serologic assays show sensitivity and specificity rates of 99.9 percent. Reference: APIC Text, 4th edition, Chapter 81

55. Which is an example of actions taken during the Study phase of the "Plan, Do, Study, Act" Performance Improvement Model? a. Identifying goals for the project b. Performing staff education sessions c. Trending and benchmarking of data collected d. Tweaking the program based on result

55. 0 Trending and benchmarking of data collected Rationale: TJC makes clear in the Infection Control and Prevention Standards that organizations should align the plan with the goal of improving infection rates. The organization must collect and display data to frontline staff about how well the organization actually achieves rate reduction. Data may include steps to increase staff influenza vaccination rates, reduce bloodstream infection and urinary catheter rates, and reduce rates of VAP. During the study phase of the cycle, data display, Benchmarking, and trending become essential. Reference: APIC Text, 4th edition, Chapter 16 - Quality Concepts CBIC Core Competency: Management and Communication

58. Which of the following must be reprocessed by high-level disinfection? a. Non-critical items b. Semi-critical items c. Critical items d. Non patient-care items

58. 0 Semi-critical items Rationale: High-level disinfection must be used for processing of semi-critical items. These are items that will-come into contact with non-intact skin or mucous membranes. High-level disinfection will inactivate all vegetative organisms but some spores may remain. High-level disinfection can be carried out by Pasteurization or by immersion in high-level disinfectants. *Reference: APIC Text, 4th edition, Chapter 106 - Sterile

59. Which of the following are CDC requirements for storing endoscopes? 1) Store them coiled in the original case 2) Store them in a bin 3) Store hanging in a vertical position to facilitate drying 4) Store in a manner that protects the scope from contamination a. 1,4 b. 1,2 c. 2,3 d. 3, 4

59. 03,4 Rationale: Endoscopes should be stored in a manner that protects them from contamination. When storing the endoscope, hang it in a vertical position to facilitate drying (with caps, valves, and other detachable components removed as per manufacturer instructions). Reference: APIC Text, 4th edition, Chapter 55

53. On September 1, there were 30 surgical patients in the hospital. Two of these were postop patients with SSIs. A total of 75 surgeries were performed in September. Six additional SS's occurred in patients who had surgery in September. What was the numerator for an incidence rate in September? a. 30 b. 6 c. 8 d. 75

6 because 2 were SSI and that took time thus they are not new... Rationale: An incidence rate is a measure of the frequency with which an event occurs in a population over a specified period of time. Incidence indicates the risk of disease in a population over a period of time. The numerator is the number of new cases of a disease during a specific time period. The denominator is the population at risk for the same time period. The incidence rate is equal to the numerator divided by the denominator and multiplied by a constant. Because there were six new cases after September 1, the numerator to calculate the incidence rate in this scenario is 6. Reference: APIC Text, 4th edition, Chapter 13

91. What is the acceptable upper limit for relative humidity in a facility to prevent fungal growth? a. 40 percent b. 5O percent c. 60 percent d. 70 percent

60 percent Rationale: Building materials contain fungal spores that have the potential to germinate in the correct conditions. Fungi require high relative humidity for growth, with most species needing relative humidity above 70 percent. Healthcare facilities should maintain relative.humidity below 60 percent to prevent the growth of fungus and to provide a comfortable patient care environment.

60. The following data on incidence rates of VAP in the Surgical ICU (SICU) were collected: • Incidence Rate of VAP NOTES 244' 244' 4.(.5s V-` Month The best interpretation of the data in the graph is: a. The number of cases of VAP was higher in January than in May b. The number of cases of VAP was the same in January and February c. Prevalence of VAP was lowest in June d. The incidence rate of VAP was highest in February

60c.o The incidence rate of VAP was highest in February Rationale: Incidence rate is calculated as the number of new cases divided by the population at risk. The graph presents the data as incidence rates, and the rate in February was higher than the rate for any other month. Reference: APIC Text, 4th edition, Chapter 11 - Surveillance CBIC Core Competency: Surveillance and Epidemiologic Investigation Reference: APIC Text, 4th edition, Chapter 20

61. A major difference between a prospective and a retrospective study is that the prospective study: a. Requires a relatively small number of subjects b. Is usually used for testing initial hypotheses c. May require a long follow-up period d. Is usually less costly

61. G May require a !Ong follow-up period Rationale: A retrospective study looks backwards and examines exposures to suspected risk or protection factors in relation to an outcome that is established at the start of the study. Retrospective studies may be undertaken in a timelier and less-expensive manner than prospective cohort studies because cases may be identified retrospectively, and at least some exposure data are often available through medical record review. A prospective cohort study watches for outcomes, such as the development of a disease, during the study period and relates this to other factors such as suspected risk or protection factor(s). The study usually involves taking a cohort of subjects and watching them over a long period. Prospective studies usually have fewer potential sources of bias and confounding than retrospective studies.

84. An IP collected the following data from the ICU for the month of March. How would she calculate the CAUTI rate the month of March? a. (6 ÷ 240) x 1,000 = 25 per 1,000 patient days b. (6 ÷ 180) x 1,000 = 33.3 per 1,000 catheter days c. (180 ÷ 240) x 1,000 = 750 per 1,000 patient days d. (240 ÷ 180) x 1,000 = 1333.3 per 1,000 catheter days

84. Q (6 +180) x 1,000 = 33.3 per 1,000 catheter days, Rationale: A rate is calculated by dividing the numerator (number of occurrences) by the denominator (number of opportunities for that occurrenee) and then multiplying by a constant. Basic formula for all types of rates:,Rate = x/y x k Where: x = The numerator, which equals the number of times the event (e.g., infections) has occurred during a specified time interval. y = The denominator, which equals a population (e.g., number of patients at risk) from which those experiencing the event were derived during the same time interval. k = A constant used to transform the result of division into a uniform quantity so that it can be compared with other, similar quantities. A whole number (fractions are inconvenient) such as 100,1,000,10,000, or 100,000 is usually used (selection of k is usually made so that the smallest rate calculated has at least one digit to the left of the decimal point) or is determined by accepted practice (the magnitude of numerator compared with denominator). There are three important aspects of the formula: 1. Persons in the denominator must reflect the same population from which the numerator was taken. 2. Counts in the numerator and denominator should coyer the same • time period: 3. At least in theory, the persons in the denominator should have been at risk of the event or occurrence. Reference: APIC Text, 4th edition, Chapter 13

90. The IP at a long-term acute care (LTAC) facility is reviewing her surveillance data. Due to the high-risk nature of the LTAC's patient population, infection control data include a large number of outliers. Which measure of central tendency is least affected by outliers? a. Proportions that include the population in the numerator and a subset of the population in the denominator b. Proportions that include the population in the denominator and a subset of the population in the numerator c. Arithmetic mean d. Median

90. ()Median Rationale: The median is the point at which 50 percent of the values fall below a middle value and 50 percent of values occur above the middle value. It is the midpoint of the observations. The median ignores extreme values and is better at indicating values close to an average. Reference: APIC Text, 4th edition, Chapter 11

92. A staff nurse informs the IP that a patient with confirmed multidrug-resistant TB requires an immediate bronchoscopy. In which of the following rooms must the procedure be performed? a. The operating room under positive pressure b. Standard procedure room in the endoscopy suite c. The ICU with direct exhaust to the outside d. Airborne infection isolation room

92. e Airborne infection isolation room Rationale: Bronchoscopy permits direct visualization of airways Using a fiberoptic bronchoscope and is used in the diagnosis and management of inflammatory, infectious, and malignant diseases of the chest. Bronchoscopy should not be performed on patients with TB unless absolutely necessary. If medically necessary, bronchoscopy should only be performed in a room that meets the ventilation requirements for an airborne infection isolation room (negative directional air flow, a minimum of-12 air exchanges per hour and direct exhaust to the outside more than 25 feet from an air intake or discharged through a 'high efficiency particulate air filtration system). Reference: APIC Text, 4th edition, Chapter 55

93. In a Gram stain procedure, Gram-positive bacteria stain purple because: a. They have a lipopolysaccharide layer in their cell wall that is decolorized with alcohol b. Their cell walls contain long-chain fatty acids that take up crystal violet easily c. They have a thick peptidoglycan cell wall that retains the primary stain during the alcohol decolorization d. Gram-staining is simple staining so the only stain used is crystal violet

93. G They have a thick peptidoglycan cell wall that retains the primary stain during the alcohol decolorization Rationale: Gram-positive cells will take up the initial crystal violet stain and will not be decolorized with alcohol because their peptidoglycan cell walls are thick enough to resist decolorization. - Reference: APIC Text, 4th edition, Chapter 24

94. Hepatitis A is diagnosed by the presence of: a. immunoglobulin G (IgG) for anti-Hepatitis D virus (HDV) b. Hepatitis A virus (HAV) lmmunoglobulin M (IgM) c. Increase in anti-Hepatitis B virus (HBV)-related to previous Hepatitis B infection d. HAV IgG

94. 0 Hepatitis A virus (HAV) immunoglobulin M (IgM) Rationale: The clinical case definition for acute viral hepatitis is (1) discrete onset ofsymptoms (e.g., nausea, anorexia, fever, malaise, or abdominal pain) and (2) jaundice or elevated serum aminotransferase levels. Because the clinical characteristics are the same for all types of acute viral hepatitis, Hepatitis A diagnosis must be confirmed by .a positive serologic test for IgM antibody to HAV, or the case must meet the clinical case definition and occur in a person who has an epidemiologic link with a person who has laboratory confirmed Hepatitis A (i.e., household or sexual contact with an infected person during the 15 to 50 days before the onset of symptoms). IgM is the first antibody built during immune response and is indicative of current disease. Reference: APIC Text, 4th edition, Chapter 97

95. When using a medication vial that is intended to be discarded after a single procedure, it is acceptable to reuse the same syringe that was just used to access that vial and give that patient medication if: a. The needle is replaced with a new needle b. The medication vial is a single-use vial c. The vial is only going to be used for that patient d. It is not acceptable to reuse the syringe

95. e It is not acceptable to reuse the syringe Rationale: While the medication in this example is expected to only be used on one patient, a healthcare worker should still use a new, sterile syringe and needle to access the medication vial the second time. This is a precaution in the event that the vial does not get discarded after use on this one patient and instead is accidentally used on another patient. References: APIC Text, 4th edition, Chapter 64

96. Which method of face-to-face instruction is a useful option when large numbers of staff must be educated over a relatively short span of time? a. Role play b. Mentoring c. Case studies d. Train the trainer

96. 0 Train the trainer Rationale: Train the trainer is an option for face-to-face institutional training when large numbers of staff must be quickly educated. Leader guides are used to train those responsible for implementing the program and for providing staff inservice and continuing education. These leader guides should be simply written, concise, and systematic. They should include curriculum goals and objectives, the course outline, instructional methods, references, and evaluation. Role playing is often used to allow the learner to experience a professional dilemma firsthand. Mentors may be used as a way to Upgrade and cross-train workforce. Case studies are viewed as an effective method to help bridge the learning gap between theory and actual practice. Reference: APIC Text, 4th edition, Chapter 3

97. An IP conducts an audit of the OR cleaning process. The action that would be most concerning would be: a. The operating room is terminally cleaned at the end of each business day b. The decontamination process starts on the floor of the OR and works upward toward the lighting c. The cleaning solutions are prepared daily d. A wet vacuum and microfiber mop head are used to clean the OR floors

97. 0 The decontamination process starts on the floor of the OR and works upward toward the lighting Rationale: At the end of each day's operating schedule, a complete terminal cleaning program should be initiated to ensure that every operating room, scrub room, and service room is properly and thoroughly cleaned. The decontamination process begins at the highest level (i.e., light tracks, ceiling fixtures) and progresses downward (i.e., shelves, tables, kick buckets, and floor). Reference: APIC Text, 4th edition, Chapter 68

99. Fifteen persons were infected with Salmonella at a picnic where 75 ate egg salad sandwiches. What was the attack rate of Salmonella among those who ate the egg salad sandwiches? a. 15 percent b. 0.20 percent c. 18 percent d. 20 percent

99e. 20 percent The attack rate for this scenario is calculated as follows: 15 ÷ 75 x 100 = 20 percent. Rationale: An attack rate is a special form of incidence rate. In fact, it is not truly a rate but a proportion. It is the proportion of persons at risk who become infected over an entire period of exposure or a measure of the risk or probability of becoming a case. It is usually expressed as a percentage and is used almost exclusively for epidemics- or outbreaks of disease where a specific population is exposed to a disease for a limited period of time. Reference: APIC Text, 4th edition, Chapter 13

5. The following blood culture result should be considered a potential contaminant: a. A positive result of coagulase-negative staphylococci from two sets, 2 days apart, without symptoms b. A positive result of S. aureus from one bottle in a patient with a temperature of 38.6°C c. A positive result of E. coli from one bottle in an afebrile patient with diarrhea d. A positive result of Candida albicans in a fungal blood culture in a patient with a urinary tract infection

A positive result of coagulase negative staphylococci from two sets, 2 days apart without symptoms Rationale: According to the CDC CLABSI criteria, common commensals (such as coagulase-negative staphylococci) meet the criteria for a CLABSI if there are two positive cultures from two or more sets of blood cultures drawn " less than 2 days apart" and the patient has symptoms (fever greater than 38°C, chills, or hypotension). CDC CLABSI criteria: Patient has at least one of the following signs or symptoms: fever (greater than 38°C), chills, or hypotension and positive laboratory results are not related an infection at another site. And the same common commensal (i.e., diphtheroids [Corynebacterium spp.. not C. diphtheriae], Bacillus spp. [not B. anthracis], Propionibacterium spp., coagulase-negative Staphylococci.[inclyding S. epidermidis], viridans group streptococci, Aerococcus spp., and Micrococcus spp.) is cultured from two or more blood cultures drawn on separate Occasions. Criterion elements must occur within a time frame that does not exceed a gap of 1 calendar day between two adjacent elements.

Describe Affinity diagrams:

Affinity diagrams gather large amounts of language data and creatively group the data based on lines of natural relationships. Data are usually collected from brainstorming or customer surveys. Interrelationship diagrams take complex, multivariable problems and graphically display all of the interrelated factors. They may also suggest cause-and-effect relationships, thereby providing focus for the team or project.

30. A preliminary microbiology report states that a patient's bloodculture grew aerobic, Gram-negative bacilli. Which of the following is the most likely genus and species of the organism: a. Enterococcus faecalis b. Bacteroides fragilis c. Acinetobacter baumanii d. Neisseria meningitides

Acinetobacter baumanii Rationale: Acinetobacter baumannii is a aerobic, Gram-negative rod (coccobacillus). It can be an opportunistic pathogen in humans, affecting people with compromised immune systems and is becoming increasingly important as an HAI. It has also been isolated from soil and water samples in the environment. Reference: APIC Text, 4th edition, Chapter 77

67. A pregnant healthcare worker is concerned because she has been assigned to take care of a patient who has cytomegalovirus (CMV) infection. How should an IP respond to this concern? a. Reassign her to another patient b. Place the patient on Contact Precautions while the healthcare worker cares for him c. Advise her that following Standard Precautions while caring for the patient will prevent transmission d. Advise her that she is likely already infected with CMV and should not worry about transmission

Advise her that following Standard Precautions while caring for the patient will prevent transmission Rationale: CHV is transmitted through saliva, urine, and blood products and organs. The CDC recommends using Standard Precautions when caring for patients with CMV, with no additional precautions recommended for pregnant HCP.N No vaccine is available for CMV and consistent adherence to Standard Precautions is the only preventive measure. References: APIC Text, 4th edition Chapter 104 - Pregnant

12. A nurse manager from a Rehabilitation Unit is requesting that aerators be installed on the sinks in the patient rooms on her unit. The IP should explain that: a. Aerators should be placed on all sinks in the ICU only. b. Aerators are more effective when combined with an ultraviolet water disinfection method c. Aerators increase the risk of HAls d. Low-flow aerators are suitable for use with this population

Aerators increase the risk of HAls Rationale: Various types of equipment and fixtures can promote the growth of water-associated pathogens. Important water reservoirs for these organisms include potable water systems and cooling towers, flush sinks, faucet aerators, hoppers and toilets, eyewash/drench shower stations, chests/ice machines, water baths used to thaw or warm blood products and other liquids, and whirlpool or spa-like baths. Faucet aerators on sinks can enhance growth of waterborne organisms. Aerators are not recommended, but if they must be used, especially in an area with immunocompromised patients, a systematic cleaning routine should be established. Reference: APIC Text, 4th edition, Chapter 115

66. Which of the following Transmission-based Precautions category requires a negative airflow room? a. Contact Precautions b. Airborne Precautions c. Droplet Precautions d. Standard Precautions

Airborne Precautions Rationale: Airborne Precautions are used to prevent transmission of infectious organisms that remain suspended in the air and travel great distances. These diseases include measles, smallpox, chickenpox, pulmonary tuberculosis, avian influenza and possibly severe acute respiratory syndrome-associated coronavirus. Patients should be placed in an airborne infection isolation (All) room with negative air pressure relative to the corridor and at least 6 to 12 air exchanges with direct exhaust of air to the outside. Monitor the air pressure daily. Keep the door shut. Reference: APIC Text, 4th edition, Chapter. 29

Describe Bacteroides fragilis?

Anaerobic gram negative rod which is normal flora of the oral cavity and GI and genital tracts.

64. Improved hydrogen peroxide contains: a. Anionic and/or nonionic surfactants b. Glutaraldehyde c. Peracetic acid d. Chlorine

Anionic and/or nonionic surfactants, Rationale: Improved hydrogen peroxide contains very low levels of anionic and/or nonionic surfactant in an acidic product that act with hydrogen peroxide to produce microbial activity. This combination of ingredients speeds the antimicrobial activity of hydrogen peroxide and clean efficiency. Reference: Chapter 31

Bioterrorism agents category A

Bacillus anthracis (anthrax) Clostridium botulinum toxin (botulism) Yersinia pestis (plague) Variola major (smallpox) and other related pox viruses Francisella tularensis (tularemia) Viral hemorrhagic fevers: Ebola, Marbug

4. What type of meningitis would be most consistent with the following cerebrospinal fluid (CSF) report result: +++++++++++++++++++++++++++++++++++++++++++++++++++++++++++++++ Glucose: decreased Protein: elevated WBC: 1,000/mm(3) +++++++++++++++++++++++++++++++++++++++++++++++++++++++++++++++ a. Bacterial b. Viral c. Fungal d. Tuberculosis

Bacterial Rationale: Culture of blood and CSF are indicated for patients with suspected invasive meningococcal disease. The CSF of- patients with untreated meningococcal meningitis is usually: cloudy pleocytosis (>WBC in CSF) with a predominance of neutrophils, low glucose, high protein levels In most of the cases, the organisms are seen on Gram stain or can be identified using latex agglutination assays. The culture is almost invariably positive as long as the samples was obtained before the administration of antibiotics. Reference: APIC Text, 4th edition, Chapter 74 - Central Nervous System Infection

126. Which of the following would be considered biomedical waste? a. Diaper soiled with feces b. Blood-tinged suctioned fluids c. Wound dressing that is not saturated d. Isolation gown worn to deliver food tray

Blood-tinged suctioned fluids Rationale: Blood and blood products, as defined in the OSHA Bloodb.orne Pathogen Standard (e.g., serum, plasma, and other components known or suspected to be contaminated with a transmissible agent) must be handled carefully. Small amounts of these materials dried on dressings or other disposable items represent an insignificant hazard once they are properly contained because of the absence of a portal of entry and a means of transmission. Bulk blood, blood-tinged suctioned fluids, excretions, and secretions are considered infectious waste because they may be splashed onto mucous membranes or the container may break and become a contaminated sharp. These fluids may be carefully poured down a drain connected to a sanitary sewer that is designed for the disposal of human waste. Personnel must follow Standard Precautions due to splash and aerosolization potential. Alternative treatment methods for inactivation or handling prior to disposal are available. Reference: Chapter 113

Bioterrorism agents category B

Burkholderia pseudomallei (melioidosis) Coxiella burnetii (Q fever) Brucella species (brucellosis) Burkholderia mallei (glanders) Chlamydia psittaci (Psittacosis) Ricin toxin (Ricinus communis) Epsilon toxin (Clostridium perfringens) Staphylococcus enterotoxin B (SEB) Typhus fever (Rickettsia prowazekii) Food- and waterborne pathogens

34. An acute care facility experiences an outbreak of Serratia marcescens bloodstream infections. After the outbreak is under control and no new cases are being reported, the IP wants to find the source of the outbreak. The most appropriate epidemiology study design to use is: a. Retrospective cohort study b. Prospective cohort study c. Case-control study d. Cross-sectional study

Case-control study Rationale: Case-control studies group people by disease status and then investigate past exposures with the objective of identifying exposures that are more common to cases than to controls. This is an appropriate study design for this example because there are existing cases, and the IP is trying to identify the exposures that are associated with the bloodstream infections. References: APIC Text, 4th edition, Chapter 10 - General

Keratitis is the inflammation of the?

Cornea—the clear dome at the front of the eye. Microbial keratitis is a true ocular emergency. Few organisms can invade the intact cornea. If the cornea epithelium is breached by trauma, insertion/removal of contact lens, or surgical intervention, organisms can enter, multiply, and destroy ocular tissue and structures. Multistate outbreaks of Fusariumand Acanthamoeba associated with contaminated contact lens solutions highlight the need for infection prevention and patient safety education in ophthalmic centers. The Fusarium outbreak was worldwide and linked to contact lens cleaning and disinfecting solution

89. Nursing Administration has announced the opening of a simulation lab at an acute care facility. The IP has been asked to develop a simulation setting dedicated to teaching infection prevention best practices. Which of the following is as example of a simulation activity? a. Creating an isolation room to practice use of Isolation Precautions b. Modeling proper hand hygiene techniques and having participants model back c. Showing an interactive video of cleaning procedures and asking participants to identify correct and incorrect elements d. Setting up a display of education materials concerning influenza vaccinations

Creating an isolation room to practice use of Isolation Precautions Rationale: The goal of simulation is to create a controlled learning environment that closely resembles the practice setting. This process facilitates use of practical and critical thinking skills on the part of the participant and serves to protect the safety of the patient. One example of a simulation room may be to create a mock isolation room. The goal may be to increase the awareness of the nursing staff regarding common infection prevention infractions that may occur during the provision of care. The simulation would enable participants to practice infection prevention activities necessary to care for patients. Reference: Chapter 3

40. After reviewing the quarterly report, the manager of the adult ICU contacts the IP for assistance to create a plan to reduce central line infections. Which of the following should the IP recommend: a. Wait for the next report to see if the rate has decreased b. Create an Intravascular Team c. Develop a multidisciplinary team to review and implement best practices d. Send a referral to Medical Affairs for peer review

Develop a multidisciplinary team to review and implement best practices Rationale: Multidisciplinary teams are a valuable tool in deploying a quality focused culture or process. Successful teams increase problem solving and efficiency, raise morale and productivity, use integrative rather than imposed solutions, increase acceptance of the solution, and tap the potential in people and their fundamental knowledge of the process. Reference: APIC Text, 4th edition, Chapter 16

122. What key strategy should the IP use to meet the administrative challenges of outpatient settings that are geographically dispersed? a. Standardized measurement and definitions for HAls consistent with those used for inpatient settings b. HAI comparison with external benchmarks for all sites c. Development of communication networks between sites, hospitals, and health departments d. Standardized infection prevention policies and procedures

Development of communication networks between sites, hospitals, and health departments Rationale: The establishment Of communication networks and collaboration at all levels of the outpatient sites is essential to implement infection prevention practices more effectively and efficiently. For outpatient sites, the use of measurement and operational definitions for HAls can be challenging due to the lack of standardized definitions. There are limited external benchmarks for HAls in ambulatory care, and HAI rates are often internally compared over time to establish a benchmark. Infection prevention policies and procedures should be specific to each setting. Reference: APIC Text, 4th edition, Chapter 48

87. Which of the following air filtration methods has the highest filtering efficiency compared to the others? a. Diffusion b. Straining c. Impingement d. Interception

Diffusion Rationale: When air enters a facility's ventilation system from the outside it passes through a series of filters that remove particulate matter. The first bank of filters has low to medium efficiency for removing particles and can include straining, impingement, and interception filtration methods. This air is then mixed with the recirculated air from the facility and filtered again with high efficiency filtration methods including diffusion filtration.

39: As a time-saving measure, hospital administrators in your facility have suggested that endocavitary probes do not need to undergo high-level disinfection because they are used with probe covers. How would you respond to this suggestion? a. Agree, because the probe cover prevents contact with mucous membranes and non-intact skin b. Disagree, because the probe covers are not 100 percent reliable c. Agree, because even without covers the probes don't contact mucous membranes and non-intact skin during use d. Disagree, because switching from high-level disinfection to low-level disinfection will not save time

Disagree, because the probe covers are not 100 percent reliable Rationale: Any patient care items that come into contact with mucous membranes and non-intact skin must undergo high-level disinfection. While probe covers are used for endocavitary probes like vaginal probes, the probe covers are prone to perforation before and during use, which can lead to contamination of the probe during the procedure.

71. While reviewing blood culture reports, you notice a note indicating that the specimen was received in the lab 3 hours after it was collected. You confirm with the microbiology supervisor. The acceptable transport time is: a. Place specimen on ice and transport within one hour of collection b. Within 24 hours of collection at room temperature c. Equal to or less than 2 hours at room temperature d. Equal to or greater than 4 hours if refrigerated

Equal to or less than 2 hours at room temperature Rationale: The reliability and value of test results depends on numerous factors. Improper collection, transport, or processing of a specimen can decrease the quality of patient care or result in unnecessary additional testing or treatment. Blood Culture bottles must be transported to the Lab within a time frame equal to or less than two hours and must be maintained at room temperature.

112. What is an advantage of experimental studies when identifying approaches to successfully interrupt the chain of infection? a. Experimental studies can identify causative disease factors retrospectively b. Experimental studies can establish association and causality if host factors such as disease susceptibility and other variables are strictly controlled c. Experimental studies avoid the need to manipulate the independent variable, and therefore ethical issues associated with exposure to disease d. Experimental studies do not require randomization, which is difficult to achieve in the healthcare environment

Experimental studies can establish association and causality If host factors such as disease susceptibility and other variables are strictly controlled Rationale: Epidemiology provides the background for interventions to reduce the transmission of infecting organisms. Selecting appropriate study design is an essential step in answering questions important to the IP. Experimental designs are always prospective. The investigator manipulates one or more factors (variables) while the others remain constant. In true experiments, randomization is used. Experimental studies can establish association and may establish causality, (when) other factors are strictly controlled. References: APIC Text, 4th edition, Chapter 10

45. The Director of the Infection Prevention Department has assigned one of her IPs to cofacilitate in a root cause analysis of an adverse event in collaboration with the Performance Improvement team. The IP plans to use process improvement tools and techniques during the analysis. Which of the following methods would best outline the possible causes of the event? a. Brainstorming b. Affinity diagrams C. Fishbone diagram d. Pareto chart

Fishbone diagram Rationale: A fishbone diagram (also called,al tree diagram or Ishikawa) allows a team to identify, explore, and graphically display all of the possible causes related to a problem to discover the root cause. See Figure PE1-1 for an example Chapter 16

Describe Neisseria meningitides?

Gram negative cocci which is normal flora of the oropharyngeal

Describe Enterococcus faecalis?

Gram positive cocci, commensal bacterium of GI tract. Formerly classified as part of the group D Streptococcus system.

1. The bacterium most likely to be transmitted from mother to infant during labor and cause neonatal sepsis is: a. Escherichia coli b. Staphylococcus aureus c. Group B Streptococcus d. Group A Streptococcus

Group B Streptococcus Rationale: In Group B Streptococcus (GBS) neonatal infections, heavy maternal colonization is associated with an increased risk for preterm labor, which in turn is a significant risk factor for neonatal infection. Intrauterine infection of the fetus therefore likely occurs via ascending spread of GBS from the vagina of a pregnant, asymptomatically colonized woman and subsequent rupture of membranes before 37 weeks' gestation. Reference: APIC Text, 4th edition, Chapter 94 - Streptococci

78. The manufacturer of a wound dressing product has notified the hospital's Purchasing Department of possible contamination of one lot of dressings. The dressings were recently approved by the Product Standardization Committee and are used in all patient care areas. Which of the following actions should the IP take? a. Instruct the Purchasing Department to remove all the manufacturer's dressings and like products from the hospital b. Notify discharged patients who were using the product while in the hospital to be alert for signs of infection and notify their physician and the Infection Prevention and Control Department c. Identify where the dressings are in the hospital, check the lot number and return them to the manufacturer, and assess the patients who used the product for signs of infection d. Notify the Health Department of the recall and provide the names of the patients who used the product

Identify where the dressings are in the hospital, check the lot number and return them to the manufacturer, and assess the patients who used the product for signs of infection Rationale: The U.S. FDA is the federal agency that is responsible for promoting public health through a number of activities, including the reasonable assurance that (1) food is safe, wholesome, sanitary, and properly labeled; (2) human drugs are safe and effective; (3) medical devices intended for human use are safe and effective; and (4) public health and safety are protected from electronic product radiation. Recalls are actions taken by a manufacturer/distributor to remove a product from the market. Recalls may be conducted on a manufacturer / distributor's own initiative (voluntary recall), by FDA request, or by FDA order. According to the FDA, a recall is a removal or correction of a product that is (1) defective; (2) a potential health risk; or (3) in violation of FDA regulations. The first step that the IP should take is to identify where the dressings are in the hospital, immediately remove them, and return them to the manufacturer. It is critical to assess the patients who used the product for signs of infection. Reference: APIC Text, 4th edition', Chapter 106

2. Which of the following is an example of the criterion of "Strength of the Association" from Hill's criteria for causation? a. In a study of the association between antibiotic exposure and development of C. difficile infection, the odds ratio was 2:3 b. In a study of the association between antibiotic exposure and development of C. difficile infection, the authors' conclusions are consistent with those of three other studies c. In a study of the association between antibiotic exposure and development of C. difficile infection, antibiotic therapy began an average of 3 weeks before C. difficile infection developed d. In a study of the association between antibiotic exposure and development of C. difficile infection, prolonged antibiotic therapy was a greater risk factor for C. difficile infection than short-term antibiotic therapy

In a study of the association between antibiotic exposure and development of C. difficile infection, the odds ratio was 2:3 Rationale: Causal associations exist when evidence indicates that one factor is clearly shown to increase the probability of the occurrence of a disease. In a causal relationship, the reduction or diminution of a factor decreases the frequency of the disease being studied. The criteria currently used for causality were developed by Austin Bradford Hill and are known as Hill's criteria. These criteria use modern epidemiological methods to determine whether a factor is causal for a given disease. Strength of association is the first criterion: The incidence of disease should be higher in those who are exposed to the factor under consideration than in those who are not exposed; that is, the stronger the association between an exposure and a disease, the more likely the exposure is to be causal. The odds ratio is a statistical measure that gives us an indication of how strongly the risk factor is associated with the disease outcome. Reference: APIC Text, 4th edition, Chapter 10

128. The local Health Department informs the IP that a nurse in the CICU has been diagnosed with measles immediately after returning from a trip to Europe. His symptoms began 2 days ago, and he last worked in the unit 9 days ago. The incubation period for measles is 8 to 12 days, and the period of contagion is 1 to 2 days prior to onset of symptoms. How should the IP follow up on this report? a. Determine the susceptibility to measles of all HCP and patients who had contact with the nurse in the past 12 days b. Place all susceptible patients who were cared for by the nurse in Airborne Isolation c. Inform Occupational Health about the infection so that they can furlough the employee for the appropriate amount of time d. The IP does not need to conduct any follow-up

Inform Occupational Health about the infection so they can furlough the employee for the appropriate amount of time Rationale: This is an appropriate follow-up in this example. Occupational Health will need to furlough the nurse for 7 days after he developed a rash. There is no risk of exposure to coworkers or patients because the employee was not in the facility during the period of contagion, so determining susceptibility of HCP and patients who had contact with the nurse and placing susceptible patients on Airborne Isolation would be unnecessary in this example. References: APIC Text, 4th edition, Chapter 86 - Measles, Mumps, Rubella; APIC Text, 4th edition, Chapter 100

56. An infant in the Neonatal ICU (NICU) has been diagnosed with Malassezia furfur fungemia. What is the most likely source of the infection? a. Intravenous lipid infusions b. A healthcare worker's false fingernails c. Commercial powdered infant formula d. Respiratory transmission from a colonized family member or healthcare worker

Intravenous lipid infusions Rationale: M. furfur is fungemia that is most often associated with lipid infusions. Reference: Chapter 10

38. You are working with the antimicrobial stewardship program in your facility to educate providers on appropriate use of antimicrobials. Which of the following would be an accurate recommendation from your group? , a. Metronidazole is an effective antimicrobial to use for MRSA skin infections b. Consistent use of broad-spectrum antimicrobials regardless of-susceptibility results will help reduce antimicrobial resistance in the facility c. It is not always necessary to maintain antimicrobial concentration in the body above the level of the minimum inhibitory concentration of the pathogen being targeted d. Cefepime is only useful for gram-negative bacterial infections

It is not always necessary to, maintain antimicrobial concentration in the body above the level of the minimum inhibitory concentration of the pathogen being targeted Rationale: The minimum inhibitory concentration (MIC) of a drug is the lowest amount that can be used that will still be effective against the pathogen. The MIC is a quantitative measure of resistance of the microbe to a drug, obtained by some antimicrobial susceptibility testing methods, such as E-test. Drugs with concentration-dependent activity are most effective when they reach a high concentration in the blood over a short period of time. The concentration of these drugs will then fall below the MIC for a period of time but they are still effective, and they can be dosed once a day. This is in contrast to drugs with time-dependent activity that should be maintained Consistently in concentrations above the MIC in the system and are administered with multiple or continuous doses. Reference: Chapter 13 & 26

65. The director of critical care has expressed a concern that there seems to be an increase in contaminated blood culture results in the ICU. An acceptable contamination level for blood cultures is: a. Less than 10 percent b.' Greater than 3 percent c. Greater than 5 percent d. Less than 3 percent

Less than 3 percent Rationale: Bacteremia is a significant cause of morbidity and mortality in hospitalized patients. Accurate and timely identification of the causative organism is imperative. Blood cultures are considered the "gold standard" in the diagnosis and treatment of bacteremia. However, the prognostic value of blood cultures is limited by contamination. Contamination, or false positive blood cultures, occurs when organisms that are not present in the blood are grown in culture. Blood culture contamination rates of less than 3 percent are desired. Reference: The Infection Preventionist's Guide to the Lab, Chapter: 3

109. Which agency or agencies has regulatory authority over some aspects of healthcare facility design, construction, and renovation related to Infection Prevention and Control? 1) Occupational Health and Safety Administration (OSHA) 2) National Institute for Occupational Safety and Health (NIOSH) 3) National Institutes of Health (NIH) 4) Centers for Disease Control and Prevention (CDC) a. 1 b. 1,2 c. 2, 3, 4 d. 1, 2, 3, 4

OSHA Rationale: OSHA is the only agency listed above that has regulatory authority over any aspect of healthcare facility design, construction, and renovation. OSHA has set regulations for engineering controls related to bloodborne pathogens and to isolation of suspected or confirmed cases of Tuberculosis in an airborne isolation room. Reference: Occupational Health and Safety Administration (OSHA

57. Which U.S. agency requires a respiratory program for HCP? a. Food and Drug Administration (FDA) b. The Joint Commission (TJC) c. Centers for Disease Control and Prevention (CDC) d. Occupational Safety and Health Administration (OSHA)

Occupational Safety and Health Administration (OSHA) Rationale: The OSHA respiratory protection standard requires that the employer implement a respiratory protection program with a qualified administrator as the person who oversees the program, including evaluating the effectiveness of the program. The standard also requires that each worker assigned to wear a respirator receive a fit test before wearing the respirator in the workplace and perform a seal check with each use. Reference: APIC Text, 4th edition, Chapter 100

Describe a Pareto chart?

Pareto charts are a series of vertical bars arranged and sorted in descending order of height from left to right with a cumulative percent line on the y-axis. Quality improvement and information technology specialists use Pareto charts to categorize data and compare units of data against the whole. Pareto charts allow a team to identify where their efforts will produce the greatest value, implying that 80 percent of benefit stems from 20 percent of causes.

What are the risk for pregnant HCP and Parvovirus B19?

Parvovirus B19 was discovered in 1975 and is the causative agent of erythema infectiosum (also known as fifth disease), a common childhood illness. This agent is of concern to pregnant HCP because B19 can cause infection of fetal red blood cell precursors and lead to severe anemia and high-output cardiac failure in the fetus, hydrops fetalis, and fetal death. Parvovirus B19 infection of patients with hemolytic anemia (e.g., sickle cell disease) can be associated with a transient aplastic crisis and with a prolonged anemia in immunodeficient hosts (e.g., HIV-infected patients, patients receiving chemotherapy). Droplet precautions are required.

What is a cohort study?

Prospective study that is easier and cheaper than a randomized controlled trial. In prospective or cohort studies, data are gathered over time. In this study design, a group of subjects with a known exposure status for the risk factor(s) of interest are followed over time to determine which of the subjects develops disease. These subjects form a cohort going through time together. Experimental studies are also prospective in nature. Data are gathered as subjects move from the present into the future while being followed up by the researcher.

25. The IP receives a call from the Food and Drug Administration (FDA) with an official request for private health information (PHI) about a patient who was admitted to the facility with botulism. How should the IP respond to this call? a. Tell the FDA officer that she cannot share PHI with the FDA due to Health Insurance Portability and Accountability Act (HIPAA) regulations b. Ask the FDA to contact the local health department to obtain information about the patient c. Provide the FDA officer with the minimum amount of information necessary related to the patient d. Transfer the call to the Risk Management Department

Provide the FDA officer with the minimum amount of information necessary related to the patient, Rationale: The FDA is a public health authority. HIPAA regulations cover disclosure to the FDA of the minimum amount of information is. necessary to prevent or control disease. References: APIC Text, 4th edition, Chapter 8

81. The heating, ventilation, and air conditioning system will need to be shut down in one patient care unit of your hospital for repairs that are anticipated to take two hours. Which of the following protocols should be followed for this event? a. Relocate all patients to other units while the shut down is in progress b. Require all patients to wear a surgical mask while the shut-down is in progress c. Relocate immunocompromised patients if necessary and provide emergency back up ventilation or portable units for other rooms d. No protocol needs to be followed as the short shut down period will not affect any patients

Relocate immunocompromised patients if necessary and provide emergency back-up ventilation or portable units for other rooms Rationale: Any shut-down of the HVAC must be coordinated with Infection Control to protect patient safety. Immunocompromised patients should be moved from the area if possible and an alternate ventilation method (emergency back-up or portable ventilation) should be used to maintain proper air flow in all rooms, including airborne isolation rooms and protective environments. Infection preventionists should be aware of the potential for a burst of spores from the HVAC when it is brought back online. In addition, critical areas will need to be given time for the appropriate number of air changes per hour to occur before they can be put back into regular use.

79. An IP has completed the investigation of an increase in postdischarge SS's following ambulatory surgical procedures. He identified some corrective measures that will require monitoring. Which of the following actions should the Infection Prevention team take? a. Monitor the cleaning of the surgical suites b. Ensure the effectiveness of the ventilation system c. Revise the facility's surveillance plan d. Ensure that patient care equipment is adequately cleaned

Revise the facility's surveillance plan Rationale: Surveillance activities should support a system that can identify risk factors for infection and other adverse advents, implement risk-reduction measures, and monitor the effectiveness of interventionli The items noted in answers A, B, and D are rarely related to the development of SSIs. The facility's surveillance program should be, revised to reflect the current risks and risk reduction measures. Reference: APIC Text, 4th edition, Chapter 11 - Surveillance

123. A hospital administrator has proposed that single-use angiography catheters be reprocessed and reused to cut costs. What is the best response to this request? a. Single-use items are not allowed to be reprocessed b. A cost-benefit analysis should be done before a decision is made c. Single-use items can be reprocessed in the facility as long as administration, Risk Management, Legal Counsel, Supply Chain Administrator, and Infection Control representatives say it is allowable d. Single-use items can be reprocessed in a third-party facility if FDA guidelines are followed

Single-use items can be reprocessed in a third-party facility if FDA guidelines are followed Rationale: The decision to reprocess and reuse a single-use item should be made by representatives from Administration, Risk Management, Legal Counsel, the supply chain administrator, and Infection Control. Third party reprocessors must be FDA-licensed to reprocess single-use devices. The reprocessing facility must maintain its own documentation, but it is recommended that the IP also review all documentation and records. Reference: APIC Text, 4th edition, Chapter 7

127. You are presenting an in-service to the Environmental Services group in your facility. Which of the following recommendations should you make related to mopping of patient rooms? a. Standard mori heads should be disinfected with a phenolic weekly b. Standard mop heads should be laundered daily c. Mop water should be changed after two hours of use d. Mop water should be changed after use on six patient rooms

Standard mop heads should be laundered daily. Rationale: Although mopping of patient rooms with a low-level disinfectant is an effective method of cleaning, there are protocols that should be followed to avoid increasing the bio-burden in rooms. Mop heads should be decontaminated by being laundered daily and mop water should be changed after use in three to four patient rooms or no more than one hour of use. Reference: CDC Guideline for Disinfection and Sterilization

Noninfectious postoperative endophthalmitis is most often associated with?

TASS, an acute, rapid onset of sterile anterior segment inflammation that mimics infectious endophthalmitis, most commonly occurs after cataract surgery. It is important to differentiate between the two because treatment, management, and patient outcomes differ. Delay in making a correct diagnosis can lead to delay in appropriate treatment and could result in vision loss or blindness. TASS cases were associated with breaches in handling, cleaning, and disinfecting of surgical instruments; introduction of contaminated solutions; contaminated intraocular lenses; toxic medications during surgery; and powder from gloves and irritants (dried blood, endotoxins, residual detergent) left on instruments

70. Which of the following patient care,units would be the best choice for conducting surveillance on wound infections with drug- resistant Gram-negative rod bacteria to prevent outbreaks? a. The Burn Unit b. The Orthopedic Medical/Surgical Unit c. The CICU d. The General Medical/Surgical Unit

The Burn Unit Rationale: Infection is the leading cause of morbidity and mortality in burn patients, despite improvements in care. Burns increase a patient's susceptibility to infection by damaging both the patient's physical and' immunological defenses. Skin is the largest organ of the body, and constitutes the first defense against infection. When burned, the integrity of the skin barrier is broken and normally sterile sites become vulnerable to microbes. Recent studies have shown an increasing prevalence of Acinetobacter, Klebsiella, and other Gram-negative rods in burn wounds. Although all of these above patient populations might be susceptible to Gram-negative rod wound infections, burn patients have the highest risk of Gram-negative wound infections, and therefore this unit is most susceptible to outbreaks 'with those organisms. Reference: APIC Text, 4th edition, Chapter:38 - Burns

135. Which of the following could be a result of a descriptive study on C. difficile in the healthcare setting? a. The average age of a patient with C. difficile b. The association between proton pump inhibitors and C. difficile c. The likelihood of having C. difficile and being placed on a specific unit d. The lack of hand hygiene by staff resulting in c. difficile acquisition

The average age of a patient with C. difficile Rationale: Epidemiological studies can be divided by levels of data and analysis. The first level of distinction is between observational and experimental studies. Observational studies involve gathering data on existing subjects with no intervention. In experimental 'studies, the researcher provides one or more interventions and determines differences before and after the intervention. The simplest type of observational study is the descriptive study. This type of study seeks to describe a population in terms of person, place, and time: who gets disease, when, and in what kind of geographical location. Examples of "person" variables include age, sex or gender, occupation, marital status, ethnicity, and health status. These studies also describe "place" variables, such as urban/rural differences, socioeconomic differences across areas, interfacility locations, and others. For these studies; time may involve an epidemic period, month, quarter, season, or periods of consecutive years or months. Descriptive studies can be used to generate rates and identify populations at risk but cannot be used to show causality. Reference: APIC Text, 4th edition, Chapter 10

37. Which of the following is not an advantage of a case-control study? a. It takes less time than a cohort study b. It requires fewer subjects c. The data are more accurate because it is prospective d. It is generally considered less expensive

The data is more accurate because it is prospective is not an advantage of a case control study. Rationale: Case-control studies collect data retrospectively. Retrospective studies are used to get information about past events and are subject to recall bias because they rely on the memory of subjects and others for information on exposure. Reference: APIC Text, 4th edition, Chapter 10

32. If chance is a likely explanation for the difference between a sample statistic and the corresponding null hypothesis population value, then: a. The difference is not statistically significant b. The sample results are not compatible with the null hypothesis c. The difference is statistically significant d. The null hypothesis can be rejected

The difference is not statistically significant Rationale: A common use of statistics is hypothesis testing. A hypothesis is a statement of expected results. Hypothesis testing uses the distribution of a known area in the normal curve and estimates the likelihood (probability) that a result did not occur by chance. Significance levels show how likely a result is due to chance. In statistics, if a result is significant, it means that it is not due to chance. If chance is a likely explanation for the difference between a sample statistic and the corresponding null hypothesis population value, then the difference is not statistically significant. Reference: Chapter 13 & 63

36. Which of the following is an example of surveillance on a process indicator? a. The incidence rate of Clostridium difficile in the Bone Marrow Transplant Unit b. The rate of hand hygiene compliance in the Bone Marrow Transplant Unit c. The number of sharp object injuries in the month of May in the Bone Marrow Transplant Unit d. The prevalence of vancomycin-resistant enterococci (VRE) in the Bone Marrow Transplant Unit

The rate of hand hygiene compliance in the Bone Marrow Transplant Unit Rationale: This is a process measure because it is measuring whether an action has taken place but not whether there is any effect of this action on outcomes. Reference: APIC Text, 4th edition, Chapter 11

35. To calculate the catheter-associated urinary tract infection (CAUTI) rate for a unit for one month, the denominator should be: a. The number of patient days for the unit for the month b. The number of admissions for the unit for the month c. The number of patients with urinary catheters for the month d. The number of urinary catheter insertions for the month

The number of patients with urinary catheters for the month Rationale: There are three important aspects of the formula in determining the CAUTI rate: (1) persons in the denominator must reflect the same - population from which the numerator was taken; (2) counts in the numerator and denominator should cover the same time period; and (3) the persons in the denominator should have been at risk of the event or occurrence (that is, number of patients with urinary catheters. Reference: APIC Text, 4th edition, Chapter 13 - Use

47. The biological indicator that was included in a steam sterilization load of non-implantable instruments has shown a positive result, as have biological indicators used in two additional follow-up tests of the sterilizer. The chemical indictor in the original load was reactive, as were the chemical indicators used in the followup tests. The log of all the runs indicates that the run conditions (temperature and time) were appropriate for the instrument load and there were no abnormalities in steam supply or electrical supply. Which of the following should be done next: a. The load should be released for use in the facility b. The sterilizer should be tested again with paired biological indicators from two different manufacturers c. The items from the load should be immediately recalled and reprocessed d. All items that were processed in that sterilizer since the time of the last negative biological indicator should be recalled and reprocessed.

The sterilizer should be tested again with paired biological indicators from two different manufacturers Rationale: Biological indictors included in steam sterilization loads may show a positive result if the sterilization parameters are not met; if there is contamination of the growth medium in the indicator; or if the indicator was not manufactured properly. With steam sterilization, the instrument load does not need to be recalled for a single positive biological indicator test, with the exception of implantable objects. The log should be checked to ensure that the sterilizer was used correctly and maintenance should be contacted to determine if there was an interruption to steam or electrical supply. If there is no indication of abnormalities, then the sterilizer should be tested again in three consecutive cycles using paired biological indicators from different manufacturers to account for a possible defective biological indicator. Reference: CDC Guideline for Disinfection and Sterilization in Healthcare Facilities, 2008 Page: 76-78, 117

68. In a published report on the risk of VRE infection in ICUs of a major teaching hospital, the authors report that the relative risk of infection is 1.9 for the Cardiac ICU (CICU) when compared to the SICU, and they conclude that a stay in the CICU is a risk factor for VRE infection. The authors have not controlled for age in their study despite the fact that their CICU patients average 20 years older than their SICU patients, and older age is a risk factor for VRE infection. The IP is concerned about the conclusions of this study because he suspects which of the following? a. The study is affected by selection bias b. The study is affected by standard error c. The, study is affected by confounding d. The study is affected by causation

The study is affected by confounding Rationale: A confounding variable is an extraneous variable (i.e., a variable that is not a focus of the study) that is statistically related to (or correlated with) the independent variable. This means that as the independent variable changes, the confounding variable changes along with it. The result is that subjects in one condition are different in some unintended way from subjects in the other condition. Confounding can lead to the assumption-that there are differences that do not really exist or to the observation that there is no difference when one truly exists. In this example, older age is associated with the risk factor of a CICU stay and with the outcome of VRE infection. The authors did not adjust for age in their study, so the IP should be concerned that the reported association is confounded by age. Reference: APIC Text,,4th edition, Chapter 10

107. The Standardized Infection Ratio (SIR) for CLABSIs in the ICU is 0.8. Which of the following is incorrect? a. There were more CLABSIs in the ICU than expected b. There were fewer CLABSIs in the ICU than expected c. There was a 20 percent reduction in CLABSIs this year from the baseline period d. Progress has been made in CLABSI reduction

There were more CLABSIs in the ICU than expected Rationale: If the SIR is less than 1, then there were fewer infections reported in 2011 than what we would have predicted given the baseline data. In other words, progress has been made since the baseline period. More infections than what would have been predicted given baseline data . Fewer infections than what would have been predicted given baseline, data Infections have been prevented since the baseline period, Infections have increased since the baseline period. 1 minus the SIR = percent reduction: SIR minus 1 = percent increase For example, the SIR of 0.80 means that there was a 20 percent reduction in 2011 from the baseline period. For example, the SIR of 1.25 means that there was a 25 percent increase in 2011 from the baseline period.

72. The epidemiologic triangle" model for disease causation does not include: a. Agent b. Host c. Time d. Environment

Time Rationale: The "epidemiological triangle" Model of disease consists of three elements: host, agent, and environment. The host is the human, and the environment consists of all external factors associated with the host. The agent may be a bacteria, virus, fungus, protozoan, helminth, or prion. In this model of dynamic interaction, a change in any component alters the existing equilibrium. Change may increase or decrease the frequency of disease. Although this model is particularly useful in the study of infectious diseases, it is also applicable to other conditions. Reference: Chapter 10

33. Noninfectious postoperative endophthalmitis is most often associated with: a. Wearing contact lens b. Toxic anterior segment syndrome (TASS) C. Conjunctivitis d. Keratitis

Toxic anterior segment syndrome (TASS) Rationale: Healthcare-associated endophthalmitis can be either noninfectious or infectious. Noninfectious endophthalmitis is an adverse event with several presenting causes, including retained lens material and other introduced toxic substances. Frequency is unknown, but occurrence is not rare. Noninfectious postoperative endophthalmitis is most often associated with TASS—an acute, rapid onset of sterile anterior segment inflammation that mimics infectious endophthalmitis. Outbreaks Of TASS have been associated with breaches in handling, cleaning, and disinfecting surgical instruments; introduction of contaminated solutions, contaminated intraocular lenses, and toxic medications during surgery; powder from gloves; and irritants (dried blood, endotoxins, residual detergent) left On instruments.

31. Urinary tract infections in the postpartum period have three important risk factors. Which of the following is not one of them: a. Induction of labor b. Tocolysis c. Cesarean delivery d. Twin births

Twin births Rationale: Urinary tract infections (LITIs) in the postpartum period have three important risk factors: cesarean delivery, tocolysis, and induction of labor. Twin births are not associated with an increased risk of UTI. Reference: Chapter 33

What is a Type 1 error?

Type I error occurs when one rejects the null hypothesis (Ho) - when it is true This is also called a false-positive result (we incorrectly conclude that the research hypothesis is true when in fact it is not).

88. The CDC has categorized bioterrorism agents according to priority. A disease that meets the criteria listed under Category B is: a. Anthrax b. Typhus c. Hantavirus d. Measles

Typhus Rationale: Agents classified by CDC as Category A are biological agents that have high potential for adverse public health impact, increased risk to national security, serious potential for large-scale dissemination, potential for public panic, social disruption, and high mortality rates. Category B agents have a moderate ease in disseminating, specific requirements for disease surveillance, moderate morbidity rates, and low mortality. Typhus (Rickettsia prowazekii) meets the Category B criteria. Reference: Bioterrorism Agents. In: Brooks K. Ready

82. An IP is asked to participate in the selection of a company to reprocess single-use devices (SUDs). Before a decision is made on which reprocessing company will be used, the IP reviews available information from a variety of resources to assist with the decision making. Which of the following activities should the IP recommend the hospital undertake? a. Talk to other infection preventionists about which sterilizing methods to use on the SUDs b. Visit reprocessing companies under consideration c. Create quality control records for the SUDs d. Identify the patients on which the reprocessed SUDs, as opposed to new devices, will be used

Visit reprocessing companies under consideration Rationale: Healthcare facilities that are considering contracting with a commercial third-party reprocessor must verify that reprocessing an SUD presents no greater risk to their patients' health and safety than using a new SUD. The decision to contract with a reprocessing company should be based on a thorough review and FDA approval of their 510(k) application. An onsite visit should be scheduled, with the opportunity to meet with personnel involved in the process, and a review of the company's policies. The visit should also include an opportunity to view the cleaning and decontamination, inspection and testing, and sterilization load preparation process, and to review quality control records. Reference: APIC Text, 4th edition, Chapter 32

42. An IP subscribes to several peer-reviewed journals. As she reviews published articles describing research findings, what question should she ask before incorporating the conclusions / findings of the article into the infection prevention program: a. Was the appropriate study design used and are the conclusions reasonable? b. Is the author well known and well published? c. Does the article state how to contact the author(s)with questions? d. Are the findings described in tables or graphs and easilyunderstandable?

Was the appropriate study design used and are the conclusions reasonable? Rationale: Many study designs, observational or experimental, are available to investigators. Understanding the advantages and disadvantages of each study design should prepare the IP to critically evaluate published research studies so as to appropriately assign value to the findings. Reference: Chapter 20

83. What is the first action HCP should take after a needlestick exposure? a. Contact the supervisor b. Contact Occupational Health c. Squeeze or milk the site d. Wash the affected area

Wash the affected area Rationale: The first action after exposure involves an immediate cleaning of the exposed/injured site. A soap-and-water hand wash should be performed to remove visible soil. Alcohol is virucidal, so an alcohol-based hand hygiene agent can be used after soap-and-water hand wash has been performed. Caustic chemicals such as bleach should)not be used in an attempt to disinfect or cleanse the skin. Squeezing or "milking" the injured site is not necessary. Once the area has been washed, evaluation by a skilled professional should besought. Reference: APIC Text, 4th edition, Chapter 81

85. The Director of Infection Prevention and Control is developing an educational program to provide annual bloodborne pathogen training to healthcare personnel in her facility. There are approximately 7,500 employees who need annual training, and her team of fi e IPs has to provide and document this training while still carrying out all other department duties. What is the best method of delivery for this training? a. Web-based training sessions b. Educational cart c. Group lecture sessions d. Role play training sessions

Web-based training sessions Rationale: Although each of the delivery methods listed has benefits, the best method in this case is web-based training. It is easy to deliver to a large number of people, flexible so that people with varied schedules can complete the training at a time that is convenient for them, and easy to track by having employees register for training through an online system. Assessments can be built into web-based training so that learners can evaluate their understanding of the training while it is in progress and demonstrate their mastery of the material at the end of the training session. Reference: APIC Text, 4th edition, Chapter,:3

23 An employee has experienced an accidental needlestick injury while providing care to a patient. All of the following lab tests would be appropriate for the source patient except: a. Human immunodeficiency virus (HIV) b. Hepatitis B antibody C. Hepatitis B surface antigen d. Hepatitis C surface antigen

b. Hepatitis B antibody Rationale: A positive Hepatitis B antibody indicates past infection or immunity via vaccination. It does not indicate active infection. Reference: APIC Text, 4th edition, Chapter 101

11. The IP is monitoring blood work in order to differentiate between bacterial or viral infection in a newly admitted ICU patient. Which of the following components of a complete blood count (CBC) should the IP examine in order to determine this information? a. Hematocrit b. White blood cell count c. Platelet (thrombocyte count) d. Red blood cell indices

White blood cell count Rationale: A white blood cell (WBC) count and differential provide information about the relative numbers (that is, the percentage) of each type of WBC. Evaluation of the WBC count can help to determine whether an illness has a bacterial or viral origin.

73. A major hurricane is forecast for your area and you need to plan for the possible contamination of potable water that is supplied to your facility. Which of the following should be included in your planning? a. You should have at least one day's worth of back up water supplies available b. You will need one liter per day of drinking water for all patients and healthcare workers c. All back up water must be stored on site for easy access d. You will need at least 25 gallons of water per patient per day to maintain care

You will need at least 25 gallons of water per patient per day to maintain care Rationale: In the event of a disaster, it is critical to ensure that there is a reliable supply of water available for drinking and for patient care activities. Individuals in the facility will need at least 2 liters of drinking water per day per person, and additional water will be needed for bathing, flushing toilets, dialysis, cooking, and other activities. It is estimated that a facility will need 25 gallons of water per patient per day to maintain these \ patient care activities. Water may be stored on-site or off-site but it must be accessible in the event of an emergency. Reference: APIC Text, 4th edition, Chapter 119

44. Which of the following scenarios would be most appropriate for immediate-use sterilization: a. The vendor brings the instrument for the procedure the morning of the surgery, which does not allow for the full sterilization process b. The instrument used for the procedure is dropped on the floor of the operating room and another instrument is not available c. The turnaround time between procedures does not allow enough time for the full sterilization process d. The OR does not have the needed instruments to meet the demand of surgeries so the instruments are flashed between procedures

]The instrument used for the procedure is dropped on the floor of the operating room and another instrument is not available Rationale: Flash sterilization is a quick-steam sterilization cycle that does not use the full sterilization cycle of exposure and dry times. Exposure may be abbreviated in gravity steam sterilizers by eliminating wrapping material or using container systems that ensure that the steam has unrestricted access to the instruments. The Association of Perioperative Registered Nurses' (AORN) Recommended Practices ,are consistent with the Association of Advancement in Medical Instrumentation (AAMI), which recommended that flash sterilization should be used only when there is an.urgent need for the items. Reference: Chapter 68 .

27. Which of the following recommendations related to disinfection and sterilization in healthcare facilities is a CDC category 1A recommendation? 1) Before use on each patient, sterilize critical medical and surgical devices and instruments that enter normally sterile tissue or the vascular system or through which a sterile body fluid flows" 2) Meticulously clean patient-care items with water and detergent, or with water and enzymatic cleaners before high-level disinfection or sterilization procedures. 3) In hospitals, perform most cleaning, disinfection, and sterilization of patient-care devices in a central processing department in order to more easily control quality. 4) Perform low-level disinfection for noncritical patient-care surfaces (e.g., bedrails, over-the-bed table) and equipment (e.g., blood pressure cuff) that touch intact skin a. 1 b. 1,3 c. 1, 2, 4 d. 1, 2, 3, 4

a. 1 Rationale: The CDC has established a system for cataloging recommendations based on the amount of data available to support the recommendation: Category 1A recommendations are strongly, supported by epidemiologic, clinical data, or experimental data from well-designed studies. Sterilization of medical instruments that will come into contact with sterile tissue or the vascular system is a Category 1A recommendation._ Reference: CDC Guideline for Disinfection and Sterilization in Healthcare Facilities, 2008 Page: 83-84

22. A pediatric patient has been diagnosed with pediculosis. What is the most appropriate follow-up to prevent it from spreading to other patients or healthcare professionals? a. Place the patient on Contact Precautions until 24 hours after appropriate treatment has been initiated b. Require all visitors and HCP who enter the room to wear a disposable scrub cap for any patient contact c. Use an insecticidal spray in the room after the patent is discharged d. Prophylactically treat all family members and anyone with close physical contact with the patient

a. Place the patient on Contact Precautions until 24 hours after appropriate treatment has been initiated Rationale: In addition to placing the patient on Contact Precautions, patient bedding, clothing, and waterproof personal items should be washed at high temperature. Reference: APIC Text, 4th edition, Chapter 96

10. Sensitivity may be defined as: a. The ability of a test to detect true positives (persons with the disease) when applied to a population with the disease b. The ability of a test to detect the true negatives (persons without the disease) when applied to a population without the disease c. The ability of a test to detect true positives (persons with disease) when applied to a population without the disease d. The percentage of persons with true positive results when the test is applied to persons without the disease

a. The ability of a test to detect true positives (persons with the disease) when applied to a population with the disease Rationale: Sensitivity and specificity are common statistical measures to describe diagnostic tests or presence of disease. Sensitivity is the ability of a test to identify true cases or persons who have the disease or health condition of interest. In other words, it is the probability of getting positive- test results among patients with disease. A high sensitivity test means that a negative result rules out the disease. Reference: APIC Text, 4th edition Chapter 13 - Use of Statistics

15. An ambulatory clinic will be transporting equipment to the local hospital for sterilization. The IP at the clinic has been asked to write a policy to ensure safe handling of the equipment by staff. The policy should include which of the following points on handling the instruments at the point of use? 1) Devices are to be cleaned before biofilm can form 2) Keep instrumentation moist to prevent bio-burden from drying 3) Instruments with lumens should be flushed with saline 4) Contaminated devices are to be placed in a sealed container to prevent exposure to staff and patients a. 1, 3, 4 b. 2, 3, 4 c. 1, 2, 4 d. ;1, 2, 3

c. 1,2,4 Rationale: Reprocessing contaminated equipment or instruments for sterilization begins at point of use. The end user is responsible for removing gross soil and debris and for rinsing items at the site of use. Instruments with lumens should be flushed with water (not saline, as salt is corrosive to most instruments). Every attempt should be made to keep instrument or equipment surfaces moist until they can be cleaned to facilitate the removal of soil. Applying enzymatic foam or gel cleaner, using wet towels placed within the set of used instruments, or presoaking used items in water or cleaning solution may also be done. Contaminated items should be placed in puncture-proof sealable containers and visibly labeled as bio-hazardous. Reference: APIC Text, 4th edition, Chapter 106 - Sterile Processing

24. The IP receives a call from a physician who is concerned that there is an outbreak of Acinetobacter baumannii, because he has cared for four patients in the past week who are infected with the organism. What is the IP's first step in responding to this call? a. Contact the lab to ask them to create an alert for any A. baumannii cases b. Contact hospital administration to request additional resources to investigate the outbreak c. Confirm that there is an outbreak by using her own surveillance data and lab records to compare the rates of Acinetobacter baumannii over the past year d. Initiate a case-control study to determine risk factors for A. baumanii infection

c. Confirm that there is an outbreak by using her own surveillance data and lab records to compare the rates of Acinetobacter baumannii over the past year Rationale: Confirming the presence of an outbreak is a key first step in an outbreak investigation. Reference: APIC Text, 4th edition, Chapter 12

28. Vaginal probes with probe covers require which type of disinfection: a. Low-level disinfection b. Intermediate-level disinfection c. High-level disinfection d. Sterilization

c. High-level disinfection Rationale: Vaginal probes are used in sonographic scanning. A vaginal probe and all endocavitary probes without a probe cover are semicritical devices because they have direct contact with mucous membranes. It is recommended that a new condom/probe cover should be used to cover the probe for each patient and, because condoms/probe covers may fail, high-level disinfection of the probe should also be performed. These medical devices should be free of all vegetative microorganisms (i.e., mycobacteria, fungi, viruses, bacteria), though small numbers of bacterial spores may be present. Reference: Chapter 31

16. A patient is admitted for a skin infection after swimming in the ocean. Which of the following organisms is the most likely cause? a. Mycobacterium marinurn b. Mycobacterium avium c. Mycobacterium leprae d. Mycobacterium tuberculosis

c. Mycobacterium marinum Rationale: M. marinum causes cutaneous lesions after exposure to swimming pools, fish tanks, or other water sources. The organisms may enter through previously unappreciated superficial nicks and abrasions. The lesions first appear as papules that later ulcerate. Because special culture conditions must be used to isolate the organism, the Microbiology Lab should be alerted if this is a diagnostic consideration. Reference: APIC Text, 4th edition Chapter 95

13. Herpes simplex virus (HSV) keratitis is suspected in an oncology patient. Aerobic eye cultures are negative to date. A single serum sample is sent for enzyme-linked immunosorbent assay (ELISA) antibody testing. The following titers are reported: HSV titer 1:128, cytomegalovirus (CMV) titer <1:8, Epstein-Barr virus (EBV) titer <1:8. These results indicate: a. Immunity to HSV b. Confirmation of acute HSV infection c. Presumptive identification of HSV infection d. Immunity to CMV and EBV

c. Presumptive identification of HSV infection HSV Type I or 2 IgM antibody for HSV infectoin: <1:10 negative 1:10 or greater is a positive Rationale: The herpesviruses are a family of eight DNA viruses that initiate acute, chronic, and latent infections of the skin, epithelial cells, lymphocytes, and neurons. These include herpes simplex type 1 (HSV-1), herpes simplex type 2 (HSV-2), varicella-zoster virus, EBV, CMV, human herpesvirus 6 (HHV-6), human herpesvirus 7 (HHV-7), and human herpesvirus 8 (HHV-8). Herpesviruses are transmitted by close intimate contact or exposure to virus containing body fluids (saliva, urine, blood, breast milk, and contaminated. respiratory or genital secretions). . Reference: APIC Text, 4th edition, Chapter 80 - Herpes Virus

19. The purpose of the antibiogram is to: a. Provide a monthly report on new and emerging antimicrobials b. Give IPs another metric to track c. Provide information on antimicrobial usage and resistance patterns in the community d. Give hospitals information needed for reporting data through the National Healthcare Safety Network (NHSN)

c. Provide information on antimicrobial usage and resistance patterns in the community Rationale: The surveillance of antimicrobial resistance is an essential first step in identifying priority areas for managing antimicrobial use from an IP perspective versus a pharmacy or cost-containment perspective. An antibiogram simplifies multiple patients' antimicrobial sensitivity information at an institution into a single number for pathogens of interest in an effort to monitor trends emerging in drug resistance. An antibiogram is a useful tool for the IP to determine the status of strategies in place to reduce MDROs. Reference: APIC Text, 4th edition, Chapter 26

26. A measles exposure from a patient in a clinic was identified and an exposure workup was initiated. A staff exposure was defined as "nonimmune HCP with more than 5 minutes of same-room contact or face-to-face contact with the index patient." Forty-eight HCP were identified as possible exposures. Of these, 44 had documented immunity to measles. Of the remaining HCP, three did not have the same room or face-to-face contact. How many HCP were at risk of developing measles because of this exposure? a. 4 b. 45 c. 1 d. 48

c.. 1 Rationale: Measles is a highly contagious febrile exanthern. In most immunocompetent individuals, measles is a self-limited condition with a distinct clinical prodrome of cough, coryza, and conjunctivitis followed by a rnorbilliform skin eruption. Measles is more severe in young, malnourished, and immunocompromised persons. Even healthy individuals may experience complications, however, such as otitis media, bronchopneumonia, encephalitis, and laryngotracheobronchitis. Because measles is so highly contagious; healthcare facilities need to be prepared to safely care for measles patients. Measles immunity (natural or vaccinated) among HCP and use of proper isolation guidelines and postexposure protocols-need to be established to minimize the potential for healthcare-associated transmission of measles. According to the definition of staff exposure, only one healthcare worker was nonimmune and had more than 5 minutes of same-room contact or face-to-face contact with the index patient. Susceptible personnel who have been exposed to an individual with measles should be furloughed (relieved from healthcare activities) from the fifth to the 21st day after exposure, regardless of whether they received measles vaccine or immunoglobulin after exposure or until 4 days after development of rash. Personnel who develop measles should be furloughed until they have had their rash for 4 days. Reference: APIC Text, 4th edition, Chapter 86

8. All of the following may be indications of a heating, ventilation, and air conditioning (HVAC) malfunction except: a. An increase in the postoperative surgical site infection (SSI) rates b. A single case of aspergillosis in a severely immunosuppressed patient c. Healthcare-associated varicella infections d. An outbreak of ventilator-associated Acinetobacter infections in the Intensive Care Unit (ICU)

d. An outbreak of ventilator-associated Acinetobacter infections in the Intensive Care Unit (ICU) Rationale: Detection and identification of certain HAls may suggest HVAC malfunction (e.g., healthcare-associated tuberculosis, single case of aspergillosis in a severely immunosuppressed patient, healthcare-associated varicella infections). Analysis of postoperative SSI rates and associated infectious agents may offer important clues to problems in the OR air system(s). HVAC systems are usually not the immediate cause of device associated HAls. Reference: APIC Text, 4th edition, Chapter 114 - Heating, Ventilation, and Air Conditioning

18 Which of the following is an example of the principle of emergency management called "mitigation?" a. Implementation of the hospital's emergency management plan during a hurricane b. Recovery efforts after a major flood has subsided c. A facility-wide bioterrorism disaster drill d. Funding a program that will provide ring vaccination of exposed people against smallpox during an outbreak

d. Funding a program that will provide ring vaccination of exposed people against smallpox during an outbreak Rationale: Disasters should be planned for and responded to using the principles of emergency management. Emergency management is composed of four principles: mitigation, preparedness, response, and recovery. Mitigation describes actions taken to decrease the potential impact of a situation. These include interventions to either prevent or reduce morbidity and mortality and ease the economic and social impact of the event on the affected community. Funding a program that will provide ring vaccination of exposed people against smallpox during an outbreak is an example of mitigation because the activity will help prevent further morbidity and mortality. Reference: APIC Text, 4th edition, Chapter 119

21. The purpose of a root cause analysis is to: a. Determine which individual made an error so that the employee may be disciplined or terminated b. Review the basic processes that are in place and then turn that review over to a unit-specific team so that they can determine how they should modify their practices c. Provide a process that requires little time or training but allows employees to identify culpability after an adverse event d. Include participants from diverse areas of the organization to delve into the cause of an 'error or systems failure and identify changes in practice and/or policy that will prevent a repeat of that error or event

d. Include participants from diverse areas of the organization to delve into the cause of an error or systems failure and identify changes in practice and/or policy that will prevent a repeat of that error or event Rationale: The root cause analysis (RCA) process takes a retrospective look at adverse outcomes and determines what happened, why it happened, and what an organization can do to prevent the situation from recurring. Risk managers commonly use the RCA to investigate major incidents, sentinel events, or errors in healthcare delivery. The RCA process avoids individual blame, considers human factors engineering, and analyzes redesign for a safer system. When conducting RCA, a multidisciplinary team discovers basic and contributing causes for what happened. The team includes frontline staff, and individuals most familiar with the situation to dig deep into the process, asking why something happens at each level of cause and effect. The entire RCA process identifies changes to a particular process or system that improves safety or reduces process error. A thorough RCA determines: (1) human and other factors; (2) the process or system involved; (3) underlying causes and effects of the process; and (4) the risks and potential contributions to failure or adverse results. Reference: APIC Text, 4th edition, Chapter 16

20. The most common organism associated with pneumonia in school-aged children and young adults is: a. Neisseria meningitidis b. Streptococcus pneumoniae c. Staphylococcus aureus d. Mycoplasma pneumoniae

d. Mycoplasma pneumoniae Rationale: Mycoplasma is Uncommon under the age of 5 but is the leading cause of pneumonia in school-aged children and young adults. It can occur during any season and occurs throughout the world.

7. The Director of the Operating Room (OR) requests that the OR surfaces be routinely environmentally cultured. The IP's best response should be: a. A schedule for routine culturing of the OR should be arranged so that each room is cultured at a set interval b. Routine culturing of the OR should be done in the absence of any epidemiologic investigations in that area C. Routine culturing should not be done because it is too expensive d. Routine environmental culturing should not be considered unless an epidemiologic investigation is being conducted

d. Routine environmental culturing should not be considered unless an epidemiologic investigation is being conducted Rationale: Microbiological environmental testing is not generally recommended. Environmental culturing can be costly and may require special laboratory procedures. Additionally, in most cases no standards for comparison exist. Because of the lack of standards, environmental testing may generate inconclusive data that could result in the implementation of unnecessary procedures or treatment. Rationale for special environmental monitoring should be carefully planned and limited to epidemiological investigations. In limited situations, "routine" environmental sampling may be indicated. Reference: APIC Text, 4th edition, Chapter 24 - Microbiology Basics

14. A patient is suspected of having Pertussis. A nasopharyngeal aspirate is collected for direct fluorescent antibody (DFA) testing and for culture eight hours after antimicrobial therapy is started. The DFA test is negative but the culture test is positive. Does this patient have Pertussis? a. No, because the DFA test is highly specific and it was negative b. No, because culture of Bordatella pertussis has very low sensitivity c. No, because all testing is unreliable if antimicrobial therapy has been initiated prior to specimen collection d. Yes, because the culture test is 100 percent specific for identification of Bordatella pertussis

d. Yes, because the culture test is 100 percent specific for identification of Bordatella pertussis Rationale: Pertussis is caused by the bacterium Bordatella pertussis. Diagnosis may occur by culture, DFA, or polymerase chain reaction testing. Culture of B. pertussis is carried out on Bordet-Gengout or Regan-Lowe media with nasopharyngeal samples and has varying sensitivity depending on specimen handling and whether antimicrobial therapy was initiated prior to specimen collection. DFA testing is very specific but not very sensitive, and PCR testing has good sensitivity and variable specificity. A negative DFA test may not mean that a patient doesn't actually have pertussis because this-test has low sensitivity and thus there is a chance of false negative tests, so a culture test must be performed to confirm DFA test results. If the culture test is negative then this cannot rule out pertussis because culture may have low sensitivity, but- if a culture test- is positive then the patient has pertussis because the 100 percent specificity of the test means that there are no false positive results., References: APIC Text, 54th edition, Chapter 71 Bordetella pertussis;

74. Patients with mycoplasma pneumonia should be cared for in which type of precautions? 1) Standard Precautions 2) Airborne Precautions 3) Droplet Precautions 4) Contact Precautions a. 1,2 b. 3, 4 c. 1,3 d. 1, 4

i,3 Rationale: Mycoplasma pneumonia is spread by respiratory droplets during close contact with a symptomatic person. In addition to Standard Precautions, Droplet Precautions are recommended for the duration of symptomatic illness.


Related study sets

C13 Section 3: RNA and Gene Expression

View Set

MLA Format & Citation Rules 1, MLA In-Text Citation Matching 3, MLA Format Quiz 2

View Set

Guide to Operating Systems - Chapter 4 (questions)

View Set

Chapt 8,9,10,11 - Consumer Behavior

View Set